You are on page 1of 160

1.

A dishonest dealer professes to sell his goods at the cost price but uses a weight of 800gm
instead of 1kg. Find his real gain percent.
A. 25%
B. 20%
C. 30%
D. None of these
Answer & Explanation
Answer: Option A.
200/800 ×100 = 25%
2. A sum of money lent out at simple interest amounts to Rs. 720 after 2 years and to Rs. 1,020
after a further period of 5 years. The sum and the rate % are
A. Rs. 500, 5%
B. Rs. 400, 15%
C. Rs. 600, 10%
D. Rs. 700, 20%
Answer & Explanation
Answer: Option C.
Amount after 2 years = Rs 720
Amount after 7 years = Rs 1020
Therefore, Interest for 5 years = Rs 300
Interest for 1 year = Rs 60
And Interest for 2 years = Rs 120
SO Principal = 720-120 = Rs 600
Also, 120 = (600*R*2)/100 = R = 10%
Amount after 2 years = Rs 720
Amount after 7 years = Rs 1020
Therefore, Interest for 5 years = Rs 300
Interest for 1 year = Rs 60
And Interest for 2 years = Rs 120
SO Principal = 720-120 = Rs 600
Also, 120 = (600*R*2)/100 = R = 10%
3. A train with 90 km/h crosses a bridge in 36 seconds. Another train 100 metres shorter crosses
the same bridge at 45 km/h. What is the time taken by the second train to cross the bridge ?
A. 61 seconds
B. 63 seconds
C. 62 seconds
D. 64 seconds
Answer & Explanation
Answer: Option D.
Train A, Speed = 90kmph
=90*(5/18)m/s = 25m/s = 25m/s, t=36s
Let length, L = x+y = time*speed = 25*36 = 900m
=800m, Speed= 45*(5/18) = (25/2) m/s
t= (Distance/Speed) = (800/(25/2)) = (1600/25) = 64 seconds
4. Ramesh travels 760 km to his home, partly by train and partly by car He takes 8 hours, if he
travels 160 km by train and the rest by car. He takes 12 minutes more, if he travels 240 km by
train and the rest by car. What are the speeds of the train and of the car?
A. Speed of car = 90 km/h, speed of train = 60 km/h
B. Speed of car = 100 km/h, speed of train = 80 km/h
C. Speed of car = 80 km/h, speed of train = 70 km/h
D. Speed of car = 100 km/h, speed of train = 90 km/h
Answer & Explanation
Answer: Option B.
Let speeds be x and y for train and car respectively.
Then 8 = (160/8) + (600/y) .....(1)
And 8(1/5) = (240/x) + ((760-240)/y) .....(2)
Solving for x and y, we get 100 and 80 km/hr.
You can also use the option straightway for such Qs.
5. Some students planned a picnic. The budget for food was Rs. 500. But, 5 of them failed to go
and thus the cost of food for each member increased by Rs. 5. How many students attended the
picnic?
A. 15
B. 25
C. 20
D. 30
Answer & Explanation
Answer: Option C.
By direction options,500/25=20 ,500/20=25
By mathematical method, the main steps are: xy = 500 …(1) and (x−5) (y+5) = 500 …(2),
From eqn. 2, x−y = 5 or y = x−5 Put in eqn 1, x(x−5) = 500 or x2-5x-500=0 ,
i.e. x = 25 and attended ones = x − 5 = 20
6. After being set up, a company manufactured 6000 scooters in the third year and 7000 scooters in
the seventh year. Assuming that the production increases uniformly by a fixed number every
year, what is the production in the tenth year?
A. 7850
B. 7650
C. 7750
D. 7950
Answer & Explanation
Answer: Option C.
You can use A.P.,Tn =a+(n-1)d ,6000=a+2d.....(1) and 7000 = a + 6d .....(2)
Eqn (2) – Eqn (1) ⇒ 1000=4d,
i.e. d = 250 and a = 6000 − 500 = 5500
T10 =5500 + 9 × 250 =7750
5. The average score of boys in an examination in a school is 71 and that of the girls is 73. The
average score of the school is 71.8. The ratio of the number of boys to that of the girls that
appeared in the examination is
A. 1 : 2
B. 3 : 2
C. 2 : 2
D. 4 : 2
Answer & Explanation
Answer: Option D.
71.8 = (71x+73y)/(x+y)
71.8 (x+ y) = 71x + 73y
0.8x = 1.2y
x:y = 12:8 which is equals to 3:2
6. The mean monthly salary paid to 75 workers in a factory is Rs. 5,680. The mean salary of 25 of
them is Rs. 5,400 and that of 30 others is Rs. 5,700. The mean salary of the remaining workers
is
A. Rs. 5,000
B. Rs. 7,000
C. Rs. 6,000
D. Rs. 8,000
Answer & Explanation
Answer: Option C
5680*75 = (5400*25+5700*30+x(75-25-30))/75
4,26,00 = 1,35,000 +1, 71,000 + 20x
X = 1,20,000/20, = 6,000
7. A sum of Rs. 25 was paid for a work which A can do in 32 days, B in 20 days, B and C in 12 days
and D in 24 days. How much did C receive if all the four work together?
A. Rs. 14/3
B. Rs. 16/3
C. Rs. 15/3
D. Rs. 17/3
Answer & Explanation
8. A man sold two steel chairs for Rs. 500 each. On one, he gains 20% and on other, he loses 12%.
How much does he gain or lose in the whole transaction?
A. 1.5% gain
B. 2% gain
C. 1.5% loss
D. 2% loss
Answer & Explanation
Answer: Option A.
CP/SP = 100/(100±x) , i.e. Total CP = 417 (500*100/200) + 568(500*100/88)≅ 985
Since CP<SP .Therefore, Profit = 100-985 = 15
P% ≅ 15/985 X 100 ≅ 1.5 %
https://wordpandit.com/
Here’s a list of few Infosys aptitude test questions
asked in interview rounds.
1. A train with 90 km/h crosses a bridge in 36 seconds. Another train 100 metres
shorter crosses the same bridge at 45 km/h. What is the time taken by the second
train to cross the bridge ?

1. 61 seconds
2. 63 seconds
3. 62 seconds
4. 64 seconds

 Click Here For Answer

2. Ramesh travels 760 km to his home, partly by train and partly by car He takes 8
hours, if he travels 160 km by train and the rest by car. He takes 12 minutes more,
if he travels 240 km by train and the rest by car. What are the speeds of the train
and of the car?

1. Speed of car = 90 km/h, speed of train = 60 km/h


2. Speed of car = 100 km/h, speed of train = 80 km/h
3. Speed of car = 80 km/h, speed of train = 70 km/h
4. Speed of car = 100 km/h, speed of train = 90 km/h

 Click Here For Answer

3. The average score of boys in an examination in a school is 71 and that of the


girls is 73. The average score of the school is 71.8. The ratio of the number of
boys to that of the girls that appeared in the examination is

1. 1:2
2. 3:2
3. 2:2
4. 4:2

 Click Here For Answer

4. Anand and Deepak started a business investing Rs. 22,500 and Rs. 35,000
respectively. Out of a total profit of Rs. 13,800, Deepak’s share is :
1. Rs. 5400
2. Rs.7200
3. Rs. 8400
4. Rs. 9600

 Click Here For Answer

5. A, B and C enter into a partnership and theirs shares are in the ratio 1/2 : 1/3 :
1/4. After two months, A withdraws half of his capital and after 10 months, a profit
of Rs.378 is divided among them. What is B’s share?
1. Rs.129
2. Rs.144
3. Rs.156
4. Rs.168

 Click Here For Answer

6. The head of a fish is 6 inches long. Its tail is as long as its head and half its
body. If its body is half of its whole length, how long is the fish?
1.16 inches
2. 24 inches
3. 32 inches
4. 48 inches

 Click Here For Answer

7. There are 3 cars moving at speeds in the ratio of . If the fastest car covers 10 km
more than the car with medium speed in 30 minutes, then the speed of the slowest
car in kmph is:
1. 30 kmph
2. 60 kmph
3. 75 kmph
4. 100 kmph

 Click Here For Answer

8. Kiran received Rs. 6000 as his share out of the total profit of Rs.9000 which he
and Arun earned at the end of one year. If Kiran invested Rs. 30,000 for 6 months,
whereas Arun invested his amount for the whole year, what was the amount
invested by Arun?
1. Rs.5000
2. Rs.6000
3. Rs.7000
4. Rs.7500

 Click Here For Answer


Answer 1.
Answer: Option D.
Train A, Speed = 90kmph
=90*(5/18)m/s = 25m/s = 25m/s, t=36s
Let length, L = x+y = time*speed = 25*36 = 900m
=800m, Speed= 45*(5/18) = (25/2) m/s
t= (Distance/Speed) = (800/(25/2)) = (1600/25) = 64 seconds
Answer 2.
Answer: Option B.
Let speeds be x and y for train and car respectively.
Then 8 = (160/8) + (600/y) …..(1)
And 8(1/5) = (240/x) + ((760-240)/y) …..(2)
Solving for x and y, we get 100 and 80 km/hr.
Answer 3.
Answer: Option D.
71.8 = (71x+73y)/(x+y)
71.8 (x+ y) = 71x + 73y
0.8x = 1.2y
x:y = 12:8 which is equals to 3:2
Answer 4.
Exp: Ratio of their shares = 22500 : 35000 = 9 : 14.
Deepak’s share = Rs. 13800 * 14/23
= Rs. 8400
Answer 5.
Exp: Ratio of initial investments = 1/2 : 1/3 : 1/4 = 6 : 4 : 3.
Let their initial investments be 6x, 2x and 3x respectively
A : B : C = (6x * 2 + 3x * 10) : (4x * 12) : (3x * 12) = 42 : 48 : 36 = 7 : 8 : 6.
Therefore, B’s share = Rs. 378 * 8/21
= Rs.144.
Answer 6.
Given, tail T = H+B/2 , where H is the head and B is the body.
Also, B = L/2 , where L is the whole length.
Here, whole length L= H+B+T
Hence, the correct choice is [4].
Answer 7.
Let the speeds of the three cars be 1.5y kmph, 2.5y kmph, and 3y kmph.
The fastest car, then, covers 3y/2 km in 30 minutes.
Therefore, the speed of the slowest car is (1.5 × 40) = 60 kmph.
Hence, choice [2] is the answer.
Answer 8.
Exp: Suppose Arun invested Rs.x. Then, Kiran : Arun = (30000 * 6) : (x * 12)
Therefore, 180000 = 6000 or x = 18000 = 7500.
12×3000 24
Question 1: A five-digit number is formed using digits 1, 3, 5, 7 and 9 without repeating any one
of them. What is the sum of all such possible numbers?
(a) 6666600
(b) 6666660
(c) 6666666
(d) None of these

Answers and Explanations

Answer: (a)
If we fix 1 at the unit’s place, the other digits can be arranged in 4! = 24 ways.
So, there are 24 numbers which have 1 at the unit’s place and sum of these numbers is 24.
Similarly, there will be 24 numbers each with 3, 5, 7 and 9 at the unit’s place.
So, the sum of all the numbers at the unit’s place will be:
24 + 24 × 3 + 24 × 5 + 24 × 7 + 24 × 9 = 24 + 72 + 120 + 168 + 216 = 600.
Similarly, the sum of the digits at the ten’s place, hundred’s place,
thousand’s place and ten thousand’s place will be 600.
So, the sum of all numbers = 600 (1 + 10 +100 +1,000 +10,000) = 6666600.
Alternative method:
The sum of all the numbers formed by the digits a1, a2, a3,……….an, without repetition of the
digits is given by:
(n-1)!(a1 + a2 + a3 + ………an) (10n -1)/9
Hence, the sum of the given numbers = 4!(1 + 3 + 5 + 7 + 9) × 11111 = 6666600.
Question 2: A man has 9 friends, 4 boys and 5 girls. In how many ways can he invite them, if
there have to be exactly 3girls in the invitees?
(a) 320
(b) 160
(c) 80
(d) 200

Answers and Explanations

Answer: (b)
Out of the 5 girls,3 girls are to be invited. It can be done in ways = 10 ways.
Now, nothing is given about the number of boys to be invited.
He can invite one boy, two boys, three boys, all the four boys or no boy.
Hence, the total number of ways of selection of boys = 24 = 16.
So, the total number of ways of invitation = 10 × 16 = 160.
Question 3: How many numbers can be formed from 1, 2, 3, 4, 5 (without repetition), when the
digit at the unit’s place must be greater than that in the ten’s place?
(a) 54
(b) 60
(c) 17
(d) 2 × 4!

Answers and Explanations

Answer: (b)
According to the question, the digit in the unit’s place should be greater than that in the ten’s
place.
Hence, if 5 is placed at the unit place, then remaining four places can be filled with any of the
four digits 1, 2, 3 and 4, hence total numbers =4!
However, if digit 4 is placed at the unit place then 5 cannot occupy the ten’s place.
Hence, the digits at the ten’s place can be 1, 2 or 3. This can happen in 3 ways.
The remaining 3 digits can be filled in the remaining three places in 3! ways.
Hence, in all we have (3 × 3!) numbers ending in 4.
Similarly, if we have 3 in the unit’s place, the ten’s place can be either 1 or 2.
This can happen in 2 ways. The remaining 3 digits can be arranged in the remaining 3 places in
3! ways.
Hence, we will have (2 × 3!) numbers ending in 3. Similarly, we have 3! numbers ending in 2 and
no number ending with 1. Hence total number of numbers= 4! + 3 × 3! + 2 × 3! + 3! = 24 + 18 +
12 + 6 = 60.
Question 4: How many numbers can be made with digits 0, 7, 8 which are greater than 0 and
less than a million?
(a) 496
(b) 486
(c) 1084
(d) 728

Answers and Explanations

Answer: (d)
Since the number is greater than 0 and less than a million, so all the numbers of single digit,
two digits, three digits, four digits, five digits and six digits formed by the digits 0, 7 and 8 will be
considered.
Now, the number of ways for selecting single digit = 2
Number of ways for selecting two digit = 2 × 3 = 6
Number of ways for selecting three digits = 2 × 3 × 3 = 18
Number of ways for selecting four digits = 2 × 3 × 3 × 3 = 54
Number of ways for selecting five digits = 2 × 3 × 3 × 3 × 3 = 162
Number of ways for selecting six digits= 2 × 3 × 3 × 3 × 3 × 3= 486
Hence, the total number of ways= (2 + 6 +18 + 54 +162 + 486) = 728.
Question 5: An intelligence agency forms a code of two distinct digits selected from 0, 1, 2, ….,
9 such that the first digit of the code is nonzero. The code, handwritten on a slip, can however
potentially create confusion, when read upsidedown-for example, the code 91 may appear as 16.
How many codes are there for which no such confusion can arise?
(a) 80
(b) 78
(c) 71
(d) 69

Answers and Explanations

Answer: (c)
The available digits are 0,1,2, …9.
The first digit can be chosen in 9 ways (0 not acceptable), the second digit can be accepted in 9
ways (digits repetition not allowed).
Thus, the code can be made in 9 × 9 = 81 ways.
Now there are only 4 digits 1, 6, 8, 9 which can create confusion.
Hence, the total number of codes which create confusion are = 4 × 3 = 12.
Out of these 12 codes 69 and 96 will not create confusion.
Hence, in total 12 – 2 = 10 codes will create confusion.
Hence, the total codes without confusion are 81 – 10 = 71
Question 1: Let n! = 1 x 2 x 3 x……….x n for integer n> 1. If p = 1! + (2 x 2!) + (3 x 3!) + ……(10
x 10!),then p+2 when divided by 11! Leaves remainder of
(a) 10
(b) 0
(c) 7
(d) 1

Answers and Explanations

Answer: (d)
If P = 1! = 1
Then P + 2 = 3, when divided by 2! remainder will be 1.
If P = 1! + 2 × 2! = 5
Then, P + 2 = 7 when divided by 3! remainder is still 1.
Hence, P = 1! + (2 × 2!) + (3 × 3!)+ ……+ (10 × 10!)
Hence, when p + 2 is divided by 11!, the remainder is 1.
Alternative method:
P = 1 + 2 × 2! + 3 × 3! + …..10 × 10!
= (2 – 1)1! + (3 – 1)2! + (4 – 1)3! + …+ (11 – 1)10!
= 2! – 1! + 3! – 2! + …..+ 11! – 10!
= 11! – 1
Hence p + 2 = 11! + 1
Hence, when p + 2 is divided by 11!, the remainder is 1
Question 2: The remainder, when (1523 + 2323) is divided by 19, is :
(a) 4
(b) 15
(c) 0
(d) 18

Answers and Explanations

Answers: (c)
an +bn is always divisible by a + b when n is odd.
Therefore 1523 + 2323 is always divisible by 15 + 23 = 38.
As 38 is a multiple of 19, 1523 + 2323 is divisible by 19.
Therefore,the required remainder is 0.
Question 3: What is the sum of all two-digit numbers that give a remainder of 3 when they are
divided by 7?
(a) 666
(b) 676
(c) 683
(d) 777

Answers and Explanations

Answers: (b)
First of all, we have to identify such 2 digit numbers.
Obviously, they are 10, 17, 24, ….94
The required sum = 10 + 17 … 94.
Now this is an A.P. with a = 10, n = 13 and d = 7
Hence, the sum is

Question 4: After the division of a number successively by 3, 4 and 7, the remainders obtained
are 2,1 and 4 respectively. What will be the remainder if 84 divides the same number?
(a) 80
(b) 75
(c) 41
(d) 53

Answers and Explanations

Answers: (d)
In the successive division, the quotient of first division becomes the dividend of the second
division and so on.
Let the last quotient be p, so the last dividend will be 7p + 4 which is the quotient of the second
division.
So, the second dividend is (7p + 4) × 4 + 1.
Applying the same logic, the number = 3 {4(7p + 4) + 1} + 2 = 84p + 53
Hence, if the number is divided by 84, the remainder is 53.
Question 5: Let N = 553 + 173 – 723. N is divisible by:
(a) both 7 and 13
(b) both 3 and 13
(c) both 17 and 7
(d) both 3 and 17

Answers and Explanations

Answers: (d)
We have N = 553 + 173 – 723 = (54 + 1)3 + (18 – 1)3 – 723
When N is divided by 3, we get remainders (1)3 + (- 1)3 – 0 = 0
Hence, the number N is divisible by 3.
Again N = (51 + 4)3 + 173 – (68 + 4)3
When N is divided by 17, the remainder is (4)3 + 0 – (4)3 = 0
Hence, the number is divisible by 17.
Hence, the number is divisible by both 3 and 17.

Question 1: Find the following sum>


1/(22 –1) +1/(42 –1) + 1/(62 –1) + …. +1/(202 –1)
(a) 9/10
(b) 10/11
(c) 19/21
(d) 10/21

Answers and Explanations


Question 2: Two men X and Y started working for a certain company at similar jobs on January
1, 1950. X asked for an initial salary of Rs. 300 with an annual increment of Rs. 30. Y asked for
an initial salary of Rs. 200 with a rise of Rs. 15 every six months. Assume that the arrangements
remained unaltered till December 31, 1959. Salary is paid on the last day of the month. What is
the total amount paid to them as salary during the period?
(a) Rs. 93,300
(b) Rs. 93,200
(c) Rs. 93,100
(d) None of these

Answers and Explanations

Question 3: Let S denote the infinite sum 2 + 5x + 9x2 + 14x3 + 20x4 + …, where | x | < 1,
then S equals
(a) {(2-x)/(1-x)3}
(b) {(2-x)/(1+x)3}
(c) {(2+x)/(1-x)3}
(d) {(2+x)/(1+x)3}
Answers and Explanations

Question 4: The infinite sum 1 + (4/7) + (9/72) + (16/73) + (25/74) + …. equals


(a) 27/14
(b) 21/13
(c) 49/27
(d) 256/147

Answers and Explanations

Question 5: Consider the set S = (1, 2, 3, …, 1000}. How many arithmetic progressions can be
formed from the elements of S that start with 1 and end with 1000 and have at least 3 elements?
(a) 3
(b) 4
(c) 6
(d) 7
(e) 8

Answers and Explanations

Answers: (d)
Let number of terms in the arithmetic progression be n, then
1000 = 1 + (n–1) d
=>(n–1) d = 999
=> n – 1 = 999/d
Since n is an integer, so n – 1 is also an integer. This means that ‘d’ is a factor of 999.
Now 999 = 33 × 37. so the total factors of 999 are 4 × 2 = 8.
Out of these 8 factors one factor is 999 and we will reject it as in that case there will be only two
terms in the
A.P. i.e. 1 and 1000, which is not possible.
Hence, ‘d’ can take 7 different values.
So, in total, 7 A.Ps. are possible

Probability is one of the trickiest (and equally fascinating)


topics in the CAT quantitative aptitude section. This topic can
feature tricky problems that test a combination of your
mathematical and reasoning skills. To help you with this topic,
we feature this set of 5 probability questions.
Question 1: One ticket is selected at random from 50 tickets numbered 0, 01, 02, ……,
49. Then, the probability that the sum of the digits on the selected ticket is 8, given that the
product of these digits is zero equals
(a) 1/14
(b) 1/7
(c) 5/14
(d) 1/50

Answers and Explanations

Explanation: (a)
Here we have
Let A be the event that sum of the digits on the selected ticket is 8, then
Let B be the event that the product of the digits is zero.
Question 2: It is given that the events A and B are such that P(A) = ¼ , P(A|B) = ½ and P(B|A) =
2/3. Then P(B|A) = 2/3. Then P(B) is
(a) 1/2
(b) 1/6
(c) 1/3
(d) 2/3

Answers and Explanations

Explanation: (c)
Question 3: A die is thrown. Let A be the event that the number obtained is greater than 3. Let B
be the event that the number obtained is less than 5. Then P (AUB) is
(a) 2/5
(b) 3/5
(c) 0
(d) 1

Answers and Explanations

Explanation:

(d)
Question 4: A pair of fair dice is thrown independently three times. The probability of getting a
total of exactly 9 twice is
(a) 1/729
(b) 8/9
(c) 8/729
(d) 8/243

Answers and Explanations

Explanation: (d)
Probability of getting a total of 9 in a single throw
4/36 = 1/9
Probability of getting a total of 9 exactly in double throw

Question 5: Two aeroplanes I and II bomb a target in succession. The probabilities of I and II
scoring a hit correctly are 0.3 and 0.2, respectively. The second plane will bomb only if the first
misses the target. The probability that the target is hit by the second plane, is
(a) 0.06
(b) 0.14
(c) 0.32
(d) 0.7

Answers and Explanations

Explanation: (c)
Let the events,
A = 1st aeroplane hit the target
B =2nd aeroplane hit the target
And their corresponding probabilities are
Extra tips for CAT Probability
AT Number System is far the most talked about topic for this exam. Majority of the students start
their CAT preparation with this topic and devote a substantial amount of time to this topic. In all
fairness, this is the right thing to do as Number System is the foundation of the subject. It teaches
you the absolute basics of the mathematics and helps you understand the subject.
Question 1: When you reverse the digits of the number 13, the number increases by 18. How
many other two digit numbers increase by 18 when their digits are reversed?
(a) 7
(b) 5
(c) 6
(d) 8
Answers: (c)

Answers and Explanations

Let the number be (10x + y), so when the digits of number are reversed the number becomes
(10y + x).
According the question, (10y + x) – (10x + y) = 18
⇒ 9 (y – x) = 18 ⇒y – x = 2
So, the possible pairs of (x, y) are
(1, 3), (2, 4), (3, 5), (4, 6), (5,7), (6, 8) and (7,9).
But we want the number other than 13.
Thus, there are 6 possible numbers, i.e., 24, 35, 46, 57, 68, 79.
So, total number of possible numbers are 6.
Question 2: A rectangular floor is fully covered with square tiles of identical size. The tiles on the
edges are white and the tiles in the interior are red. The number of white tiles is the same as the
number of red tiles. A possible value of the number of tiles along one edge of the floor is :
(a) 10
(b) 12
(c) 14
(d) 16
Answers and Explanations

Answers: (b)
Let the rectangle has x and y tiles along its length and breadth respectively.
The number of white tiles
W = 2x + 2(y – 2) = 2 (x + y – 2)
And the number of red tiles = R = xy – 2 (x + y – 2)
Given that the number of white tiles is the same as the number of red tiles
⇒ 2 (x + y – 2) = xy – 2 (x + y – 2)
⇒ 4 (x + y – 2) = xy
⇒ xy – 4x – 4y = –8
⇒ (x – 4) (y – 4) = 8 = 8 ×1 or 4 × 2
⇒m – 4 = 8 or 4 ⇒m = 12 or 8
Therefore.the number of tiles along one edge of the floor can be 12
Question 3: X persons stand on the circumference of a circle at distinct points. Each possible
pair of persons, not standing next to each other, sings a two-minute song one pair after the other.
If the total time taken for singing is 28 minutes, what is X?
(a) 5
(b) 7
(c) 9
(d) None of these

Answers and Explanations

Answer (b)

Question 4: Each family in a locality has at most two adults, and no family has fewer than 3
children. Considering all the families together, there are more adults than boys, more boys than
girls, and more girls than families. Then the minimum possible number of families in the locality is
: (a) 4 (b) 5 (c) 2 (d) 3

Answers and Explanations

Answer (c)
Given, Number of adults > Number of boys > Number of girls > Number of families.
Going back from the choices, let us start with the least value given in the choices.
Since the minimum possible number of families has been asked.
In choice (c), Number of families = 2
⇒Number of girls > 3, Number of boys > 4 and Number of adults > 5
But two families together can have a maximum of 4 adults.
Therefore, the number of families is not equal to 2.
In Choice (d). Number of families = 3.
Therefore, the Number of (girls) > 4. Number of (boys) > 5 and Number of (adults) > 6
3 families can have 9 children and a maximum of 6 adults.
Hence, the minimum number of families = 3.
Question 4: Each family in a locality has at most two adults, and no family has fewer than 3
children. Considering all the families together, there are more adults than boys, more boys than
girls, and more girls than families. Then the minimum possible number of families in the locality is
:
(a) 4
(b) 5
(c) 2
(d) 3

Answers and Explanations

Answers: (d)
Given, Number of adults > Number of boys > Number of girls > Number of families.
Going back from the choices, let us start with the least value given in the choices.
Since the minimum possible number of families has been asked.
In choice (c), Number of families = 2
⇒Number of girls > 3, Number of boys > 4 and Number of adults > 5
But two families together can have a maximum of 4 adults.
Therefore, the number of families is not equal to 2.
In Choice (d). Number of families = 3.
Therefore, the Number of (girls) > 4. Number of (boys) > 5 and Number of (adults) >6
Question 5:The owner of a local jewellery store hired 3 watchmen to guard his diamonds, but a
thief still got in and stole some diamonds. On the way out, the thief met each watchman, one at a
time. To each he gave 1/2 of the diamonds had then, and 2 more besides. He escaped with one
diamond. How many did he steel originally?
(a) 40
(b) 36
(c) 25
(d) None of these

Answers and Explanations


How to solve CAT Number System questions?
Question 1:A report consists of 20 sheets each of 55 lines and each such line consists of 65
characters. This report is reduced onto sheets each of 65 lines such that each line consists of 70
characters. The percentage reduction in number of sheets is closest to:
(a) 20%
(b) 5%
(c) 30%
(d) 35%

Answer and Explanation

Option a
No. of Characters in one line = 65
No. of characters in one sheet = No. of lines × No. of characters per line = 55 × 65
Total number of characters = No. of sheets × No. of characters in one sheet = 20 × 55 × 65 =
71500
If the report is retyped –
New sheets have 65 lines, with 70 characters per line
No. of characters in one sheet = 65 × 70
Number of pages required,

Hence, 16 pages will be required if report is retyped.


Hence, reduction of (20 – 16) = 4 pages
% reduction is = (4/20) x 100 = 20%
Question 2: The number of votes not cast for the PNC Party increased by 25% in the National
General Election over those not cast for it in the previous Assembly Polls, and the PNC Party lost
by a majority twice as large as that by which it had won the Assembly Polls. If a total 2,60,000
people voted each time, how many voted for the PNC Party in the previous Assembly Polls?
(a) 1,10,000
(b) 1,50,000
(c) 1,40,000
(d) 1,20,000
Answer and Explanation

Option c
Total Votes = 2,60,000
Let x voters voted against the party in the Assembly Poll.
Then votes in favour = 260000 – x
Therefore, majority of votes by which party won in previous poll = 260000– x – x = 260000 – 2x
Next year votes against the PNC party increase by 25%
So, votes against the party in general election = 1.25 x
And votes polled in favour of the party = total votes – votes against = 260000 – 1.25x
Therefore, majority of votes by which party lost in general election
= 1.25x – (260000 – 1.25x) = 2.5x – 260000
It is given that, PNC Party lost by a majority twice as large as that by which it had won the
Assembly Polls, Therefore
2.5x – 260000 = 2(260000 – 2 x)
⇒ 2.5x – 260000 = 2 260000 – 4x
⇒ 6.5x = 3260000⇒x == 1,20,000
Therefore, votes polled by the voters for the party in Assembly Polls for previous year
= (2,60,000 – x) = (2,60,000 – 1,20,000) = 1,40,000.
Question 3: 2/5th of the voters promise to vote for A and the rest promised to vote for B. Of
these, on the last day 15% of the voters went back of their promise to vote for A and 25% of
voters went back of their promise to vote for B, and A lost by 200 votes. Then, the total number
of voters is:
(a) 10000
(b) 11000
(c) 9000
(d) 9500

Answer and Explanation

Option a
Let x be the total number of voters
Voters promised to A = 2/5 x
Voters backed out = 15% of 2/5 x
Voters promised to B = 3/5 x
Voters backed out = 25% of 3/5 x
Total Number of votes for A = 2/5 x – 15% of 2/5 x + 25% of 3/5 x
Question 4: A person who has a certain amount with him goes to market. He can buy 50
oranges or 40 mangoes. He retains 10% of the amount for taxi fares and buys 20 mangoes and
of the balance, he purchases oranges. Number of oranges he can purchase is:
(a) 36
(b) 40
(c) 15
(d) 20

Answer and Explanation

Option d
The person can buy 50 oranges or 40 mangoes.
Let the price of one orange be Rs. x
Total amount the person has = Rs. 50x
40 mangoes cost 50x, So one mango costs 1.25x
10% of the total amount is retained for taxi fare = 10% of 50x = 5x
20 mangoes bought for 20 x 1.25x = 25x
Money left with the person = 50x – (Taxi fare) – (Mangoes cost)
= 50x – 5x – 25x = 20x
One Orange was for Rs. x, Therefore, 20 oranges can be bought with Rs. 20 x
Thus, the person bought 20 oranges.
Question 5: Forty per cent of the employees of a certain company are men and 75% of the men
earn more than Rs. 25,000 per year. If 45% of the company’s employees earn more than Rs.
25,000 per year, what fraction of the women employed by the company earn Rs. 25,000 or less
per year?
(a) 2/11
(b) 1/4
(c) 1/3
(d) 3/4

Answer and Explanation


Option d
Let the total number of employees in the company be x
Then the number of men and women be 0.4x and 0.6x respectively.
75% of men earn more than Rs. 25000 => 0.75 x 0.4 x = 0.3 x
Total number of employees earning more than Rs. 25000 = 45% x = 0.45 x
Number of women earning more than Rs. 25000 =
Total employees earning more than Rs. 25000 – total number of Men earning more than Rs.
25000
= 0.45 x – 0.30 x = 0.15 x
Number of the women earning Rs. 25000 or less = 0.60 x – 0.15 x = 0.45 x
Fraction of the women employed by the company who earn Rs. 25000 or less
(0.45x/0.60x) = 45/60 = ¾
Alternate Solution:
Let there are 40 men and 60 women in the company. Now out of 40 men, 75% i.e. 30 earn more
than Rs 25000 and 45% of the total employees i.e. 45 employees earn more than Rs 25000.
Hence, there are
45 – 30 = 15 women who earn more than Rs25000. So, 60 – 15 = 45 women earn less than Rs
25000.
Hence, the required fraction = 45/60 = ¾
Extra tips for CAT Percentage Questions

Question 1: Alphonso, on his death bed, keeps half his property for this wife and divides the rest
equally among his three sons : Ben, Carl and Dave. Some years later, Ben dies leaving half his
property to his widow and half to his brothers Carl and Dave together, sharing equally. When
Carl makes his will, he keeps half his property for his widow and the rest he bequeaths to his
younger brother Dave. When Dave dies some years later, he keeps half his property for his
widow and the remaining for his mother. The mother now has Rs. 1,575,000.What was the worth
of the total property?
(a) Rs. 30 lakh
(b) Rs. 8 lakh
(c) Rs. 18 lakh
(d) Rs. 24 lakh

Answer and Explanation

Option d
Let the total property of the Alphonso be Rs.x.
After Alphonso’s death, money possessed by the family members would be
Wife = x/2, Ben = x/6, Carl =x/6, Dave =x/6
After Ben’s death, money possessed by each of them would be
Alphonso’s wife = x/2, Ben = 0,Ben’s wife = x/6, Carl = x/6 + x/24 = 5x/24, Dave = x/6 + x/24 =
5x/24
After Carl’s death, money possessed by them
Alphonso’s wife has x/2, Ben has 0, Ben’s wife has x/12, Carl has 0, Carl’s wife has 5x/48, Dave
has
5x/24 + 5x/48 = 15x/48
After Dave’s death, money possessed by them is:
Alphonso’s wife has x/2 + 15x/96 = 63x/96, Ben has 0, Ben’s wife has x/12, Carl’s has 0, Carl’s
wife has
5x/48, Dave has 0 and Dave’s wife has 15x/96
Now, given that 63x/96 = 1575000
x= 2400000
Alternative Method:
You can also solve this question by using options.
If we take total amount to be Rs 2400000, then after Alphonso’s death, the money with the family
members will be:
Alphonso’s wife = 1200000, Ben = Carl = Dave = 400000, Ben will leave 100000 each for Carl
and Dave.
So, Carl and Dave have 50000 each, Carl will leave 250000 for Dave, so Dave has 750000.
Dave left 750000/2 = 375000 for his mother, so his mother has 1200000 + 375000 = 1575000,
which is given in the question. hence option 4 is the answer.
Question 2: Two oranges, three bananas and four apples cost Rs. 15. Three oranges, two
bananas and one apple cost Rs. 10. I bought 3 oranges, 3 bananas and 3 apples. How much did
I pay?
(a) Rs. 10
(b) Rs. 8
(c) Rs. 15
(d) Cannot be determined

Answer and Explanation

Option c
It is given that
2O + 3B + 4A = 15 …..(1)
3O + 2B + A = 10…….(2)
The answer to this question seems to be cannot be determined as we are given three variables
but we can form two equations only. But the question is not asking about the individual price of 3
oranges, 3 bananas and 3 apples but it asks the cost of 3O + 3B + 3A. For that, if we add the two
equations, we get
5O+5B+5A=25
O+B+A=5
Therefore 3O +3B+3A = 3×5 = 15
Question 3: A watch dealer incurs an expense of Rs. 150 for producing every watch. He also
incurs an additional expenditure of Rs. 30,000, which is independent of the number of watches
produced. If he is able to sell a watch during the season, he sells it for Rs. 250. If he fails to do
so, he has to sell each watch for Rs. 100.If he produces 1500 watches, what is the number of
watches that he must sell during the season in order to breakeven, given that he is able to sell all
the watches produced?
(a) 500
(b) 700
(c) 800
(d) 1000

Answer and Explanation

Option b
Total cost to produce 1500 watches = (1500 × 150 + 30000) = Rs. 2,55,000
Let he sells x watches during the season, therefore
number of watches sold after the season = (1500 – x)
∴Revenue earned on the sale of 1500 watches
= 250 × x + (1500 – x) × 100 = 150x + 150000
Now, break-even is achieved if production cost is equal to the selling price.
∴ 150x + 150000 = 2,55,000 ⇒x = 700
Question 4: A stockist wants to make some profit by selling sugar. He contemplates about
various methods. Which of the following would maximize his profit?
1. Sell sugar at 10% profit.
2. Use 900 g of weight instead of 1 kg.
3. Mix 10% impurities in sugar and selling sugar at cost price.
4. Increase the price by 5% and reduce weights by 5%.
(a) I or III
(b) II
(c) II, III and IV
(d) Profits are same

Answer and Explanation

Option b
We will solve this question by taking all the cases one by one.
In the first case it is given that the profit is 10%.
For second case, let the CP of 1 kg of sugar be Rs. 100
Then CP of 900 g of sugar= (100/1000 )x 900 = Rs. 90
Hence, profit % in Case II= [{(100-90)/90}x100] = 11.11%
For case III, If he adds 10% impurity then his CP for 1 kg
= {(100/1100) x 100} = Rs. 90.90
Hence, profit % in Case III = [{(100-90.90)/90.90} x 100] = 10.01%
and in the last case, If he reduces weight by 5%
Then cost price of 950 g = {(100/1000) x 950} = Rs. 95 and SP = Rs. 105
Hence, profit % in Case IV = {(105 – 95 )/95} X 100 = 10.52%
Hence, the profit is maximum in second case.
Question 5: A dealer offers a cash discount of 20% and still makes a profit of 20%, when he
further allows 16 articles to a dozen to a particularly sticky bargainer. How much per cent above
the cost price were his wares listed ?
(a) 100%
(b) 80%
(c)75%
(d) 66%

Answer and Explanation

Option a
Let the CP of the article be Rs. x, since he earns a profit of 20%, hence SP = 1.2x.
It is given that he is selling 16 articles to a dozen, so he a incurs loss by
selling 16 articles at the cost of 12 articles [loss = {(16-12)/16} x 100 = 25%]
∴ His selling price = SP × 0.75
Now SP × 0.75 = 1.2 x⇒ SP = (1.2/0.75)x = 1.6x.
This SP is arrived after giving a discount of 20% on MP.
Hence, MP = (1.6/0.8)x = 2x
It means that article has been marked 100% above the cost price.
Alternative Method:
Let the cost price = Rs 100. Since the profit is 20%, so the SP = Rs 120.
This SP = Rs 120 is arrived after giving a discount of 20%, i.e. MP = 120/0.8 = Rs 150.
Now he is selling 16 goods to a dozen, so his loss in this case = {(16-12)/16} x100 = 25%.
It means that Rs 150 were arrived after losing 25%. Hence the actual MP = 150/0.75 = Rs 200.
Hence, he has marked the MP 100% above the CP.

Extra tips for CAT Profit and Loss Questions


Question 1: There is a leak in the bottom of the tank. This leak can empty a full tank in 8 hr.
When the tank is full, a tap is opened into the tank which intakes water at rate of 6 L per hour and
the tank is now emptied in 12 hr. What is the capacity of the tank?
(a) 288 L
(b) 36 L
(c) 144 L
(d) Cannot be determined

Answer and Explanation

Answers: (c)
Let the tap can fill the tank in x hours.
The rate of leak per hour = (1/8)
and rate of leak when the tap is on = (1 / 12).
Hence we have 1/8 – 1/x = 1/12 1/x = 1/8 – 1/12 = 2/48 = 1/24
i.e. the tap can fill the tank in 24 hours.
The tap fills at 6 litres per hour, so it will fill 6 × 24 = 144 litres in 24 hours.
Thus the capacity of the tank is 144 litres.
Question 2: A water tank has three tap: A, B and C. A fills 4 buckets in 24 min, B fills 8 buckets
in 1 hr and C fills 2 buckets 20 min. If all the taps are opened together, a full tank is emptied in 2
hr. If a bucket contains 5 L water, what is the capacity of the tank?
(a) 120 L
(b) 240 L
(c) 180 L
(d) 60 L

Answer and Explanation

Answers: (b)
The capacity of the bucket is 5 litres
So, tap A fills 20 litres water in 24 min.
Tap B fills 40 litres water in 60 min and tap C fills 10 litres water in 20 min
Hence work done by all the taps together in 2

hours Thus the capacity of the


tank is 240 litres.
Question 3: One man can do as much work in one day as a woman can do in 2 days. A child
does one-third the work in a day as a woman. If an estate owner hires 39 pairs of hands—men,
women and children in the ratio 6 : 5 : 2 and pays them in all Rs. 1, 113 at the end of the day’ s
work, what must the daily wages of a child be, if the wages are proportional to the amount of
work done ?
(a) Rs 14
(b) Rs 5
(c) Rs 20
(d) Rs 7

Answer and Explanation


Answers: (d)
Ratio of number of men, women and children = 6 : 5 : 2 and the total number of men, women and
children = 39.
Therefore the number of men = 18, women = 15 and children = 6.
Ratio of work done by men:women: children = 6 : 3 : 1
∴ Ratio of work done by 18 men, 15 women and 6 children
= (18 × 6) : (15 × 3) : (6 × 1) = 108 : 45 : 6
Hence Rs.1113 would be divided in this ratio.
That makes Rs756 for men, Rs 315 for women and Rs42 for children. Hence 6 children earn
Rs42 in a day. So the daily
wage of a child is equal to 42/6 = Rs7
Hence, option d.
Question 4: Two typists undertake to do a job. The second typist begin working one hour after
the first. Three hours after the first typist has begun working, there is still (9/20) of the work to be
done. When the assignment is completed, it turns out that each typist has done half the work.
How many hours would it take each one to do the whole job individually?
(a) 12 hr and 8 hr
(b) 8 hr and 5.6 hr
(c) 10 hr and 8 hr
(d) 5 hr and 4 hr

Answer and Explanation

Answers: (c)
Let the first typist takex hours and second typist take y hours to do the whole job.
So 3 hours work of first typist and 2 hours work of second typist combined = 11/20
3/x + 2/y = 11/20 …(1)
Also it has been given that finally both have done the same amount of work.
x/2-y/(2 ) = 1 …(2)
From (1) and (2), we get x = 10 hours and y = 8 hours.
Hence, option c.
Question 5: A group of men decided to do a job in 8 days. But since 10 men dropped out every
day, the job got completed at the end of the 12th day. How many men were there at the
beginning?
(a) 165
(b) 175
(c) 80
(d) None of these

Answer and Explanation

Answers: (a)
Let initially there were x men and each man can do 1 unit work in one day. As the work was to be
completed in 8 days, so the total work is 8x.
Now every day, 10 workers dropped out. Hence we have
8x = x + (x – 10) + ( x – 20) ……. + (x – 110) = 12 (x – 55)
=>8x = 12x – 660
=>4x = 660
=>x = 165
Hence, option a.
Question 1: The sides of a triangle are 5, 12 and 13 units. A rectangle is constructed, which is
equal in area to the triangle, has a width of 10 units. Then, the perimeter of the rectangle is:
(a) 30 units
(b) 36 units
(c) 13 units
(d) None of these

Answer and Explanation

Answer: (d)
5,12 and 13 are Pythagoras triplets. Hence the triangle so formed will be a right angled triangle.
Greatest side is the hypotenuse which is 13, and with 12 as height and 5 as base.
Area of triangle = ½ × Base × Height = ½ × 12 × 5 = 30 sq units.
Area of rectangle = Area of Triangle = 30 sq units Area of rectangle = length × breadth = 30.
Given breadth or width = 10 units Therefore, length × 10 = 30 =>length = 3 units.
Perimeter of the rectangle = 2 × (length + breadth) = 2 × (3 + 10) = 2 × 13 = 26 units.
Hence the perimeter of the rectangle is 26 units.
Question 2: PQRS is a square. SR is a tangent (at point S) to the circle with centre O and TR=
OS. Then, the ratio of area of the circle to the area of the square is:

(a) π/3
(b) 11/7
(c) 3/ π
(d) 7/11

Answer and Explanation

Answer: (a)
Let x m be the side of the square => PQ = QR = RS = SP = x m
Let r be the radius of the circle => OS = OT = r m
Then we have following –
Now Given TR = OT = r m
OR = OT + TR
=r+r
= 2r
Area of the circle is πr2
Area of Square is x2
Now in triangle RSO,
OS2 + SR2 = OR2
r2 + x2 = (2r)2 = 4r2
x2 = 3r2
So area of Square PQRS= x2 = 3r2
Therefore the ratio of area of the circle to the area of the square is = πr2 /3r2 =π/3
Question 3: Which one of the following cannot be the ratio of angles in a right-angled triangle?
(a) 1: 2: 3
(b) 1: 1: 2
(c) 1: 3: 6
(d) None of these

Answer and Explanation

Answer: (c)
We can solve the answer by using the options provided to us. We know that the sum of angles of
a triangle is 180˚ and in a right angled triangle the largest angle is 90˚. Using these two facts we
shall evaluate the options given –
Option (a) 1: 2: 3 => Angles are x, 2x and 3x. Sum = 6x = 180
=> x = 30. Angles are 30, 60 and 90. Hence it is a right angle triangle.
Option (b) 1: 1: 2 => Angles are x, x and 2x. Sum = 4x = 180
=> x = 45. Angles are 45, 45 and 90.
Hence it is a right angle triangle.
Option (c) 1: 3: 6 => Angles are x, 3x and 6x.
Sum = 10x = 180 => x = 18.
Angles are 18, 54 and 108. Here none of the angles is of 900, therefore this cannot be the ratio of
angles of a right angled triangle.
Question 4: A wooden box (open at the top) of thickness 0·5 cm, length 21 cm, width 11 cm and
height 6 cm is painted on the inside. The expenses of painting are Rs.70. What is the rate of
painting per square centimeter?
(a) Re. 0·7
(b) Re 0·5
(c) Re 0·1
(d) Re 0·2

Answer and Explanation

Answer: (c)
The dimensions given to us are from outside, and the box is painted from inside, we need to find
the dimensions of the inside of the box.
Thickness = 0.5 cm
The thickness needs to be subtracted for getting internal dimensions.
length = 21 cm
Inside length = length – 2 × thickness = 21 – 2 × 0.5 = 21 – 1 = 20 cm
Breadth = 11 cm
Inside breadth = Breadth – 2 × thickness = 11 – 2 × 0.5 = 11 – 1 = 10 cm
Height = 6 cm
Inside height = Height – thickness = 6 – 0.5 = 5.5 cm
(Note: it is an open box, so thickness from height is subtracted only once)
Total Area to be painted = Area of 4 side walls + Area of the base
Area of 4 walls = 2 × height × (length + breadth) = 2 × 5.5 × (20 + 10) = 330 sq cm
Area of Base = length × breadth = 20 × 10 = 200 sq cm
Total area to be painted = 530 sq cm
Cost of painting = Rs. 70
Cost per sq cm = 70/530 = 0.13 or Re 0.1 per sq. cm (approx)
Question 5: From a circular sheet of paper with a radius 20 cm, four circles of radius 5 cm each
are cut out. What is the ratio of the uncut to the cut portion?
(a) 1: 3
(b) 4: 1
(c) 3: 1
(d) 4: 3

Answer and Explanation

Answer: (c)
Following is the representation of the question given above,

Area of Circle is π R2
Area of sheet of paper with radius 20 cm π (20)2 = 400 πcm2
Area of cut portion = Area of 4 circles of radius 5 cm = 4π(5)2 = 100 πcm2
Area of uncut portion = Area of sheet of paper – Area of cut portion
Therefore, Area of uncut portion = 400 π-100 π = 100 πcm2
Therefore, Ratio of area of uncut portion to area of cut portion
300 π:100 π= 3:1
Directions (Q. Nos. 1-4): Consider the following graph where the prices of timber are given for
the period 1997-2003. The prices for plywood and sawn timber are given in Rs./m3 while the
price of logs is given in Rs. /tonne. Assume 1 tonne in equal to 1000 kg and one cubic meter of
logs weighs 800 kg.
Timber Product Prices

Question 1: Which product had the largest percentage increase in price per cubic meter over the
7 yr period?
(a) Sawn timber
(b) Logs
(c) Plywood
(d) Cannot be determined
Question 2: The maximum increase in price per cubic meter for any product between two years
was
(a) Rs. 2500
(b) Rs. 3000
(c) Rs. 2000
(d) Rs. 4125
Question 3: In 2003, the total sales of the company measured in cubic meters was made up of
40% plywood, 30% sawn timber and 30% logs.The average realisation per cubic meter in 2003
was closest to
(a) Rs. 16500
(b) Rs. 13500
(c) Rs. 14480
(d) Rs. 18000
Question 4: In 2004, the prices of plywood, sawn timber and logs went up by 5%, 1% and 10%,
respectively and the total sales were made up of 40% plywood, 30% sawn timber and 30% logs.
The average realisation per cubic meter in 2004 was closest to
(a) Rs. 15000
(b) Rs. 16150
(c) Rs.14500
(d) Rs.18500
Answers and Explanations

Answer 1: (b)
Plywood = {(20000-14000)/14000} x 100 = 42.84%
Swan Timber = {(16000-10000)/10000} x 100 = 60%
Logs = {(7000-4000)/4000} x 100 = 75%
NOTE: For logs, the percentage increase calculation would not be affected by the change
in units.
Thus, the correct option is (b)
Answer 2: (b)
Solution: 1 cu. m. = 800 Kg
1 tonne= 1000 Kg
1/1000 tonne = 1 Kg
800 Kg = 0.8 tonne
1 cu. m = 0.8 tonne
Remember, for logs, we simply multiply every value with 0.8. Simple logic is if price for 1000 kg is
4000 then price for 800 kg cannot be more than 4000.
Therefore, the new table with new log prices will be

It can be seen that the maximum increase in price per cubic meter is for Plywood for the year
1998 from 1997 which is Rs 3000. Correct option is (b)
Answer 3: (c)

Let total cubic meters sold be 10 cu. m.


Plywood sold = 4 cu. m
Sawn Timber sold = 3 cu. m
Logs sold = 3 cu. m.
Average= {(16000 x 3) + (20000 x 4) + (5600 x 3)}/10 = Rs. 14480
Correct option is (c)
Answer 4: (a)

In 2004, the prices of Sawn timber, Plywood and logs increased by Rs 160, Rs 1000 and Rs 560
respectively. So the new prices of Sawn timber, Plywood and logs become Rs 16160, Rs 21000
and Rs 6160respectively
Let total cubic meters sold be 10 cu. m.
Plywood sold = 4 cu. m
Sawn Timber sold = 3 cu. m
Logs sold = 3 cu. m.
Hence average price
= [{(16160 x 3) + (21000 x 4) + (6160 x 3)}/10]
= [{48480 +84000+18480}/10]
= 150960/10 = Rs. 15096
Among the options, option (a) is the closest and is answer.

Directions (Q. Nos. 1-4): The scatter diagram shows the number of students passing in the high
school’ examination in the given years from the four houses of a public School.
Question 1: The average number of students for each house who have passed in the given
years nearest to
(a) 59
(b) 52
(c) 63
(d) 56
Question 2: The performance for which of the following houses is the best?
(a) Pearl
(b) Ruby
(c) Topaz
(d) Sapphire
Question 3: For which of the following houses is the percentage change in the results maximum
for any year over the previous year?
(a) Topaz
(b) Pearl
(c) Sapphire
(d) Ruby
Question 4: The number of students keeps on increasing by 50 every years. In 1998, there were
250 students. For which of the following years is the performance best in the school?
(a) 1998
(b) 2000
(c) 1999
(d) Cannot be determined

Answers and Explanations


From above diagram we can make the following table
:

Answer 1: (a) The total number of students = 718 in all years. Average = 718/12 = 59.8 (approx)
Therefore, closest option is (a)
Answer 2: (d) By the total we can see Sapphire has more students passed in 3 years as
compared to others. Correct option is (d)
Answer 3: (b) In Pearl house, students passing the exam in 1999 is 75 whereas in 1998 is 45.
Hence the percentage change
= 75−4545×100=3045×100=66.675−4545×100=3045×100=66.6which is highest. Correct
option (b)
Answer 4:
(a)

From the data, we can see that the percentage of students passing the exam is highest in 1998.
Hence, the best performance is in 1998. Correct option is (a)
Directions for questions 1 to 6: Study the following information carefully and answer the
given questions.
Eight friends Qadir, Rakesh, Satish, Tarun, Vishwas, Wazir, Yusuf and Zakir are sitting around a
circular table facing the centre. 3 friends are males and the rest are females. The following
conditions are followed by the group:
• Vishwas sits second to the right of his wife.
• Satish sits third to the right of Vishwas.
• Wazir sits second to the right of her husband Zakir.
• Zakir is not an immediate neighbour of Vishwas’s wife.
• Tarun is a male
• Yusuf is not an immediate neighbour of Vishwas.
• Rakesh sits second to the right of Qadir.
• No two males are immediate neighbors of each other.
Question 1: What is the position of Tarun with respect to Zakir?
(a) First to the left
(b) Immediately to the right
(c) Second to the left
(d) Third to the right
Question 2: Which of the following statements regarding Satish is definitely correct?
(a) Satish is one of the male members of the group
(b) Both the immediate neighbours of Satish are females
(c) Satish sits fourth to the left of T
(d) Wazir is an immediate neighbour of Satish
Question 3: Vishawas’s wife is:
(a) Q
(b) Y
(c) R
(d) T
Question 4: Rakesh is seated immediately between:
(a) Two Males
(b) Two females
(c) One male and one female
(d) Cannot be determined
Question 5: Who sits next to Zakir?
(a) Rakesh and Vishwas
(b) Rakesh and Wazir
(c) Rakesh and Satish
(d) Satish and Vishwas
Question 6: How many people sit between Rakesh and Tarun?
(a) None
(b) One
(c) Two
(d) Three

Answers and Explanations: Click the down arrow to expand

Explanation for Questions 1 to 6:


STEP 1: First fix the position of Vishwas’s wife and subsequently place Vishwas and Satish.
STEP 2: Since Wazir sits second to the right of her husband Zakir, the only possible position for
Zakir is to the left of Satish.
STEP 3: Since Tarun is a male and thus cannot be a neighbor of Vishwas. Step 4: Fill in rest of
the positions easily.
Answer 1: (d)
Tarun is third to the right of Zakir.
Answer 2 : (d)
We can see from the figure that option (d) is correct.
Answer 3 : (b)
In the question, we are given that Vishwas sits second to the right of his wife.
From the figure, we can deduce that Vishwas’s wife is Yusuf.
Answer 4 : (a)
From the figure, we can deduce that Rakesh is seated between two males.
Answer 5 : (c)
From the figure we can see that Rakesh and Satish are the neighbors of Zakir.
Answer 6 : (d)
From the figure, we can deduce that three people are seated between Rakesh and Tarun.

Directions for Questions 1 to 5: Bharat, Mahesh, Tarun, Rakhi, Kailash, Hans, Dev are seven
people from different professions and they are travelling to Pune from either through a plane or a
train or a bus. Each of them has a different profession, that of dancer, cook, artist, fashion
designer, sweeper, peon and poet.
• Two of seven are using the plane, three are using the train and the rest are using the bus.
• Bharat, the cook, is not in the Bus.
• The artist is the only other person who travels the same way as Bharat.
• Mahesh and Hans are not in the Train and they are the poet and the sweeper respectively.
• Tarun is a Fashion Designer.
• Dev is neither the peon nor the artist.
• Kailash travels the same way as the dancer.
Question 1: Dev’s profession is
(a) Dancer
(b) Cook
(c) Sweeper
(d) Fashion Designer
Question 2: The people who travel through train are:
(a) Dev ,Kailash , Rakhi
(b) Dev , Hans , Tarun
(c) Hans , Kailash, Tarun
(d) Dev ,Kailash , Tarun
Question 3: Which of the following is correct?
(a) Rakhi – plane – Peon
(b) Rakhi –plane – Artist
(c) Dev – Bus – Dancer
(d) Kailash – Bus – Sweeper
Question 4: Which of the following pairs travel using the plane?
(a) Bharat, Dev
(b) Bharat, Kailash
(c) Bharat,Tarun
(d) none
Question 5: Who is the artist?
(a) Dev
(b) Hans
(c) Rakhi
(d) Data inadequate

Answers and Explanations: Click the down arrow to expand

Explanation for Questions 1 to 5


We can draw the following table using the information provided to us:

Bharat is not in the bus.


The artist travels the same way as Bharat travels, and only two of them travel this way (means
they either use the bus or the plane). Since bus is ruled out, the artist and Bharat travel on Plane.
Now the artist can be Rakhi or Kailash.
But Kailash travels with the Dancer (Dev) and Bharat is not a dancer. Thus, Rakhi travels with
Bharat and is the artist. Kailash is the peon.
Since the plane is taken by Bharat and Rakhi, Mahesh and Hans take the bus as they do not
take the train. Since they take the bus, and Kailash and Dev have to be together, it means
Kailash and Dev are joined by Tarun on the train. Thus, Kailash and Dev take the train.

Answer 1: (a)
The answer can be found from the tables above.
Answer 2 : (d)
Using the tables above, we can find the answer.
Answer 3 : (b)
Using the table above: Rakhi – Aero plane – Artis
Answer 4 : (d)
None of the pairs are correct.
Answer 5 : (c)
The answer can be found from the tables above.
Directions (Qs. 1-3) : Answer the questions based on the following information.
Recently, Bosch Babu spent his winter vacation on Hawaai Island. During the vacation, he visited
the local casino where he came across a new card game. Two athletes, using a normal deck of
52 playing cards, play this game. One athlete is called the ‘broker’ and the other is called the
‘athlete’. First, the athlete picks a card at random from the deck. This is called the base card. The
amount in rupees equal to the face value of the base card is called the base amount. The face
values of ace, king, queen and jack are ten. For other cards the face value is the number on the
card. Once the ‘athlete’ picks a card from the deck, the ‘broker’ pays him the base amount. Then,
the ‘broker’ picks a card from the deck and this card is called the top card. If the top card is of the
same suit as the base card, the ‘athlete’ pays twice the base amount to the ‘broker’. If the top
card is of the same colour as the base card (but not the same suit), then the ‘athlete’ pays the
base amount to the ‘broker’. If the top card happens to be of a different colour than the base
card, the ‘broker’ pays the base amount to the ‘athlete’.
Bosch Babu played the game four times. First time he picked eight of clubs and the ‘broker’
picked queen of clubs. Second time, he picked ten of hearts and the ‘broker’ picked two of
spades. Next time, Bosch Babu picked six of diamonds and the ‘broker’ picked ace of hearts.
Lastly, he picked eight of spades and the ‘broker’ picked jack of spades. Answer the following
questions based on these four games.
Question 1 : The initial money Bosch Babu had (before the beginning of the game sessions)
was Rs. X. At no point did he have to borrow any money, the minimum possible value of X
(a) 100
(b) 16
(c) 8
(d) 24
Question 2 : If Bosch Babu stopped playing the game when his gain would be maximized,
What would be the gain in Rs.?
(a) 12
(b) 4
(c) 20
(d) 16
Question 3 : If the final amount of money that Bosch Babu had with him was Rs. 100, the initial
amount he had with him is
(a) 96
(b) 8
(c) 120
(d) 4

Answers and Explanations

Answers: Questions (1-3): The information given in the question can be presented in the
tabular form as under

Answer 1 : (C)
The minimum amount has to be Rs. 8 as he was left with a negative balance of Rs.8 after the
first round.
Answer 2 : (A)
The maximum gain of Bosch Babu is Rs. 12 which is same after second and third round of game.
Answer 3 : (A)
Let initial amount of money which he had with him be Rs. X. Bosch has gained Rs 4 by playing
the game.
X + 4 = 100 or X = Rs. 96.
https://wordpandit.com/logical-reasoning-level-3-set-23/
Directions (Qs.1-4): Refer to the following information and answer the questions that follow:
‘Kem-kem” is an island in the South Pacific. The inhabitants of “Kem-kem” always answer any
question with two sentences. One of which is always true and the other is always false.
Question 1: You are walking on a road and come to a fork. You ask the inhabitants Krishna,
Balram and Sita, “Which road will take me to the village?”
Balram says, “I am married to Sita. Take the left road.”
Krishna says, “I never speak to strangers, I am new to these parts”.
Sita says, “I am married to Krishna. He is not new to this place.”
Which of the following is true?
(a) Right road takes you to the village
(b) Left road takes you to the village
(c) Sita is married to Balram
(d) None of these
Question 2: You want to speak to the chief of the village. You question to three inhabitants,
Amar, Akbar and Antony. Only Akbar is wearing red shirt
A. Amar says, “I am not Akbar’s son. The chief wears a red shirt.”
B. Akbar says, “I am Amar’s father, Antony is the chief.”
C. Antony says, “The chief is one among us. I am the chief.”
Who is the chief ?
(a) Amar
(b) Akbar
(c) Antony
(d) None of them
Question 3: You find that your boat is stolen. You question three inhabitants of the island and
they reply as follows:
Ajinkya says, “I didn’t do it. Dhoni didn’t do it.”
Dhoni says, “I didn’t do it. Krishna didn’t do it.”
Krishna says, “I didn’t do it. I don’t know who did it.”
Who stole your boat? ”
(a) Krishna
(b) Dhoni
(c) Ajinkya
(d) None of them
Question 4: There is only one pilot on the island. You interview three men-Shantanu, Aman and
Shivam. You also notice that Shantanu is wearing a cap.
A. Shantanu says, “I am the priest. On this island, only priests can wear caps.”
B. Shivam says, “Aman’s father is the pilot. Aman is not the priest’s son.”
C. Aman says, “I am the priest’s son. Shantanu is not the priest.”
Which of the following is true?
(a) Aman is the priest
(b) Shantanu is the pilot
(c) Shivam is the pilot
(d) Aman is not Shantanu’s son

Answers and Explanations

Answer 1: (b)
“Left road takes you to the village” is the correct statement.
Answer 2: (b)
It is clear that Akbar is the chief. All other options go contrary to the information.
Answer 3: (b)
Let us assume first part of the first statement to be true then second part will be false. If we relate
these results with second and then third statements assuming one part to be true and other false.
Then we will find that answer is Dhoni. On ‘the contrary if we assume first half of the first
statement to be false and second to be true then there will be two answers Ajinkya and Krishna,
which cannot be true. Hence the correct answer is Dhoni.
Answer 4: (b)
Option (b) is the only correct statement.
Directions for questions no. (1-5): A circular field, with inner radius of 10 meters and outer
radius of 20 meters, was divided into five successive stages for ploughing. The ploughing of each
stages was handed over to a different farmer.
1. Farmers are referred to by following symbols: S1, S2, S3, S4, S5.
2. The points between different stages of project are referred to by the following symbols: A1, A2,
A3, A4, A5, not necessarily in the same order.
3. Farmer S4 was given the work of the fourth stage.
4. Stage 3 finished at point A1, and the work of which was not given to farmer S1.
5. Farmer S3 was given work of stage ending at point A5.
6. Farmer S5 was given the work of ploughing stage starting at point A4.
7. The stage from point A5 to point A3 was not the first stage.
Question 1: Which were the starting and the finish points of stage 2?
A. A2 and A5
B. A5 and A3
C. A3 and A1
D. A5 and A4
E. A3 and A2
Question 2: For which farmer was A2 a finishing point?
A. S1
B. S2
C. S3
D. S4
E. S5
Question 3: Which stage was ploughed by farmer S5?
A. First
B. Second
C. Third
D. Fourth
E. Fifth
Question 4: Which was the starting point for Farmer S3?
A. A2
B. A3
C. A4
D. A1
E. None of above
Question 5: Which was the finish point for farmer S2?
A. A1
B. A2
C. A3
D. A4
E. A5

Answers and Explanations

Solution:
For questions 1 to 5: Stage 3 finished at point A1 and the work in the stage 4 was done by S4.
It is given that one of the stage starts with A5 and ends with A3, and that stage cannot be the
stage 1 (Condition 4), stage 3 (Condition 6), stage 4 (Condition 6) or stage 5 (Condition 4).
Hence, stage 2 starts and ends with point A5 and A3 respectively. Therefore, S3 has ploughed in
stage 1 (Condition 7). So stage 5 will start and end with A4 and A2 respectively (Condition 3) So
the work of stage 2 and stage 3 will be done by S1 and S2 respectively(Condition 6). Based on
the inferences, the following table can be formed:

Answer 1: (B)
Answer 2: (E)
Answer 3: (E)
Answer 4: (A)
Answer 5: (A)
Direction for the questions: (1- 5:) India invited representatives of seven countries to visit the
Taj Mahal. The 7 representatives, John, Nichola, Bob, Tony, Tesla, Bill and Queen are from
Hongkong, Beijing, New York, Sydney, Paris, Tokyo, Berlin(not in same order). 7 cars were sent
to receive them from the airport. Each vehicle is of a different color and each representative
selects a color of his choice. Three representatives are females. John likes the green color
vehicle but John is not from Hong Kong or New York. The Chinese likes the grey colored vehicle
and is a female. Tesla, who is the sister of Bill, is Sydney and likes the Red colored vehicle.
Queen is from Berlin and likes the Orange colored vehicle. Bill, the wife of Bob is from New York
and likes yellow color vehicle. Nichola likes Grey color vehicle and Bob likes Blue color and the
one who likes Pink is from Paris.
Question 1: Who is from Beijing?
(a) Bill
(b) John
(c) Nichola
(d) Tony
Question 2: Which of the following is a group of girls?
(a) Tesla ,Bill , John
(b) Nichola,Tesla, Bill
(c) Nichola,Queen ,Bill
(d) Cannot be determined
Question 3: Bob is from
(a) Paris
(b) Beijing
(c) Hong Kong
(d) None
Question 4: The representative from Tokyo is:
(a) John
(b) Queen
(c) Queen
(d) Bob
Question 5: Which of the following is correct?
(a) Blue – Bob – Beijing
(b) Pink – Tony – Sydney
(c) Red – Bill – New york
(d) Pink – Tony – Paris

Answers and Explanations


Explanation for questions 1 to 5: From the given information, we can easily build the table of
representatives and their favorite colored vehicles. Key points: Bill, the wife of Bob is from New
york and likes yellow colored vehicle Quella, who is the sister of Bill, lives in Sydney and likes the
Red colored vehicle

The one who is from Beijing likes the grey colored vehicle and is a female, and from table 1,
Nichola is from Beijing. Thus, Bill, Quella and Nichola are the females.
Answer 1 : (c)
Nichola is from Beijing.
Answer 2 : (b)
Nichola, Bill and Quella are Girls.
Answer 3 : (c)
Bob is from Hongkong
Answer 4 : (a)
John is the representative from Tokyo.
Answer 5 : (d)
The combination Pink –Tony –Paris is correct.

Question 1: I walk to a town at 312 kmph, rest there for 45 minutes and ride back
at 712kmph. Find the distance to the town, if the total time spent by me is 6 hrs 37
min.
[1] 14 km
[2] 7km
[3] 5 km
[4] 8 km
Answer & Explanation
Answer:
Option: 1

Explanation:
Let the distance be D.
Therefore, D3.5+4560+D7.5=63760⇒D=14cm

Question 2: A train overtakes two persons who are walking in the same direction in
which the train is going, at the rate of 2 kmph and 4 kmph and passes them
completely in 9 seconds and 10 seconds respectively. The length of the train (in
metres) is
[1] 45
[2] 54
[3] 50
[4] 72

Answer & Explanation

Answer:
Option: 3

Explanation:
Let the length of train be x km and its speed be y km/hr.
Then, xy−2=960×60andxy−4=1060×60
=> 9y – 3600x = 18 and 10y -3600x =40
=> y- 400x = 2 and y – 360x =4

Therefore, 40x = 2 or x m
Question 3: A man reaches his office 30 min late, if he walks from his home at 3 km
per hour and reaches 40 min early if he walks at 4 km per hour. How far is his office
from his house?
[1] 7km
[2] 14 km
[3] 5 km
[4] 3 km

Answer & Explanation

Answer:
Option: 2

Explanation:
Time gained = 30 + 40 = 70 min =7060hrs.
Let the distance be x km.
Therefore, x3−x4=70×160⇒x=14 km

Question 4: Raju, walking at the rate of 6 kmph, covers a certain distance in three
hours. In how much time will Raju cover this distance running at the speed of 18
kmph?
[1] 1 hour
[2] 3 hours
[3] 60 hours
[4] 22 hours

Answer & Explanation

Answer:
Option: 1

Explanation:
Let the distance be X.
=>Distance = Speed x Time taken = 6 x 3 = 18 km.
Now, speed = 18 km/hr.
=>Time taken = Distance/Speed = 18/ 18 = 1 hour.

Question 5: Two cyclists start together to travel to a certain destination, one at the
rate of 4 kmph and the other at the rate of 5 kmph. Find the distance if the former
arrives half an hour after the latter.
[1] 2 km
[2] 10m
[3] 10000m
[4] 1 km

Answer & Explanation

Answer:
Option: 3

Explanation:
Let the distance be X km. Now Time= distance/ speed.
=>Time for the first rider = X/14.
Similarly, time for the second cyclist = X/5.
Given that: X/14 – X/5 =1/2
=> X= 10 km = 10000 m

question)Ampee, Bumpee, Chumpee, Dumpee and Pumpee are five ducks. Initially
they had 1, 2, 3, 4, 5 eggs, not necessarily in that order. They laid 1, 2, 3, 4, 5 eggs
(not necessarily in that order) and finally had 4, 5, 6, 7, 8 eggs (not necessarily in
that order) in the end. Further information regarding them is given:

1. Initially Dumpee had 2 eggs and ended up with 7 eggs at the end.
2. Bumpee laid 3 eggs and did not end in 8 eggs.
3. Pumpee did not lay 1 egg or 2 eggs.
4. Ampee ended with the number of eggs Chumpee started with.

Question: How many eggs the duck that started with 3 eggs lay
Show Answer
2

Question: How many eggs the duck that ended with 6 eggs lay

Show Answer
1

Question: What is the name of the duck that laid 1 egg

[1] Ampee
[2] Bumpee
[3] Chumpee
[4] Either Ampee or Bumpee
Show Answer
Option #3
Question: The duck that ended with maximum number of eggs started with how many
number of egg

Show Answer
4

Syllogisms
Directions for Q. No. 1 to 25:
In each of the following questions two statements are given and these
statements are followed by two conclusions numbered (A) and (B). You have
to take the given two statements to be true even if they seem to be at variance
from commonly known facts. Read the conclusions and then decide which of
the given conclusions logically follows from the two given statements,
disregarding commonly known facts. Give answer: (A) If only (A) conclusion
follows (B) If only (B) conclusion follows (C) If either (A) or (B) follows (D) If
neither (A) nor (B) follows and (E) If both (A) and (B) follow.
1. Statements:
All the poets are goats. Some goats are trees.
Conclusions:
A. Some poets are trees.
B. Some trees are goats.

2. Statements:
Some mangoes are yellow. Some tixo are mangoes.
Conclusions:
A. Some mangoes are green.
B. Tixo is a yellow.

3. Statements:
Some ants are parrots. All the parrots are apples.
Conclusions:
A. All the apples are parrots.
B. Some ants are apples.

4. Statements:
Some pearls are jewels. Some jewels are ornaments.
Conclusions:
A. Some jewels are pearls.
B. Some ornaments are jewels.
5. Statements:
Some hens are cows. All the cows are horses.
Conclusions:
A. Some horses are hens.
B. Some hens are horses.

6. Statements:
Some papers are pens. All the pencils are pens.
Conclusions:
A. Some pens are pencils.
B. Some pens are papers.

7. Statements:
All the actors are girls. All the girls are beautiful.
Conclusions:
A. All the actors are beautiful.
B. Some girls are actors.

8. Statements:
All the flowers are leaves. Some leaves are birds.
Conclusions:
A. Some birds are flowers.
B. Some leaves are flowers.

9. Statements:
All the windows are doors. No door is a wall.
Conclusions:
A. Some windows are walls.
B. No wall is a door

10. Statements:
All cups are books.
All books are shirts.
Conclusions:
A. Some cups are not shirts.
B. Some shirts are cups.

11. Statements:
Some cows are crows. Some crows are elephants.
Conclusions:
A. Some cows are elephants.
B. All crows are elephants.
12. Statements:
All the pencils are pens. All the pens are inks.
Conclusions:
A. All the pencils are inks.
B. Some inks are pencils.

13. Statements:
All the trucks are flies. Some scooters are flies.
Conclusions:
A. All the trucks are scooters.
B. Some scooters are trucks.

14. Statements:
All buildings are chalks. No chalk is toffee.
Conclusions:
A. No building is toffee
B. All chalks are buildings

15. Statements:
All cars are cats. All fans are cats.
Conclusions:
A. All cars are fans.
B. Some fans are cars.

Directions for Q. No. 16 to 25: In each of the following questions two


statements are given which are followed by four conclusions (A), (B), (C) and
(D). Choose the conclusions which logically follow from the given statements.
16. Statements:
No door is dog. All the dogs are cats.
Conclusions:
A. No door is cat.
B. No cat is door.
C. Some cats are dogs.
D. All the cats are dogs.
Options:
a. Only (B) and (D)
b. Only (A) and (C)
c. Only (C) and (D)
d. Only (C)
e. All the four
17. Statements:
All green are blue. All blue are white.
Conclusions:
A. Some blue are green.
B. Some white are green.
C. Some green are not white. D. All white
D. All white are blue.
Options:
a. Only (A) and (B)
b. Only (A) and (C)
c. Only (A) and (D)
d. Only (B) and (D)

18.Statements:
All men are vertebrates. Some mammals are vertebrates.
Conclusions:
A. All men are mammals.
B. All mammals are men.
C. Some vertebrates are mammals.
D. All vertebrates are men.
Options:
a. Only (D)
b. Only (B)
c. Only (C)
d. Only (A)
e. Only (A) and (C)

19.Statements:
All the phones are scales. All the scales are calculators.
Conclusions:
A. All the calculators are scales.
B. All the phones are calculators
C. All the scales are phones.
D. Some calculators are phones.
Options:
a. Only (A) and (D)
b. Only (C) and (D)
c. Only (B) and (D)
d. Only (A) and (B)
e. Only (A) and (C)

20. Statements:
Some cars are scooters. No scooter is cycle.
Conclusions:
A. No car is cycle.
B. No scooter is car.
C. Some cars are cycles.
D. Some scooters are cars.
Options:
a. None of the four.
b. All the four.
c. Only (A) and (D)
d. Only (D)
e. Only (B) and (D)

21. Statements:
Some tables are T.V. Some T.V. are radios.
Conclusions:
A. Some tables are radios.
B. Some radios are tables.
C. All the radios are T.V.
D. All the T.V. are tables.
Options:
a. Only (B) and (D)
b. Only (A) and (C)
c. Only (D)
d. Only (A) and (D)
e. None of the four.

22. Statements:
Some pens are books. Some books are pencils.
Conclusions:
A. Some pens are pencils.
B. Some pencils are pens.
C. All pencils are pens.
D. All books are pens.
Options:
a. Only (A) and (C)
b. Only (B) and (D)
c. All the four
d. None of the four
e. Only (A)

23. Statements:
All the goats are tigers. All the tigers are lions.
Conclusions:
A. All the goats are lions.
B. All the lions are goats.
C. Some lions are goats.
D. Some tigers are goats.
Options:
(a) All the four
(b) Only (A), (B) and (C)
(c) Only (A), (C) and (D)
(d) Only (B), (C) and (D)

24. Statements:
All the books are pencils. No pencil is eraser.
Conclusions:
A. All the pencils are books.
B. Some erasers are books.
C. No book is eraser.
D. Some books are erasers.
Options:
a. Only (C)
b. Only (A) and (C)
c. Only (A) and (B)
d. Only (B) and (C)
e. Only (C) and (D)

25. Statements:
All the research scholars are psychologists. Some psychologists are scientists.
Conclusions:
A. All the research scholars are scientists.
B. Some research scholars are scientists.
C. Some scientists are psychologists.
D. Some psychologists are research scholars.
Options:
a. Only (C) and (D)
b. None of the four
c. All the four
d. Only (C)
e. Only (B) and (D)

Directions for Q. No. 26 to 34: In each of the following questions there


are three statements. Which are followed by three or four
conclusions. Choose the conclusions which logically follow from the
given statements.
26. Statements:
All the locks are keys. All the keys are bats. Some watches are bats.
Conclusions:
A. Some bats are locks.
B. Some watches are keys.
C. All the keys are locks.
Options:
a. Only (A) and (B)
b. Only (A)
c. Only (B)
d. Only (A) and (C)

27. Statements:
Some keys are staplers. Some staplers are stickers. All the stickers are pens.
Conclusions:
A. Some pens are staplers.
B. Some stickers are keys.
C. No sticker is key.
D. Some staplers are keys.
Options:
a. Only (A) and (B)
b. Only (B) and (D)
c. Only (B) and (C)
d. Only (A) and (D)
e. either (B) or (C)

28. Statements:
Some questions are answers. Some answers are writers. All the writers are
poets.
Conclusions:
A. Some writers are answers.
B. Some poets are questions.
C. All the questions are poets.
D. Some poets are answers.
Options:
a. Only (A) and (B)
b. Only (A) and (D)
c. Only (A) and (C)
d. Only (B) and (D)

29. Statements:
Some envelops are gums. Some gums are seals. Some seals are adhesives.
Conclusions:
A. Some envelopes are seals.
B. Some gums are adhesives.
C. Some adhesives are seals.
D. Some adhesives are gums.

Options:
a. Only (C)
b. Only (A)
c. Only (B)
d. Only (D)

30. Statements:
All the papers are books. All the bags are books. Some purses are bags.
Conclusions:
A. Some papers are bags.
B. Some books are papers.
C. Some books are purses.
Options:
a. Only (A)
b. Only (B) and (C)
c. Only (A) and (B)
d. Only (A) and (C)

31. Statements:
Some rats are cats. Some cats are dogs. No dog is cow.
Conclusions:
A. No cow is cat.
B. No dog is rat.
C. Some cats are rats.
Options:
a. Only (A)
b. Only (A) and (B)
c. Only (A) and (C)
d. Only (B) and (C)
e. Only (C)

32. Statements:
All the books are papers. Some papers are journals. Some journals are
calendars.
Conclusions:
A. Some journals are books.
B. Some calendars are papers.
C. Some books are journals.
D. Some books are calendars.
Options:
a. Only (A)
b. Only (B)
c. Only (C)
d. Only (D)
e. None of the four

33. Statements:
All the bottles are boxes. All the boxes are bags. Some bags are trays.
Conclusions:
A. Some bottles are trays.
B. Some trays are boxes.
C. All the bottles are bags.
D. Some trays are bags.
Options:
a. Only (C) and (D)
b. Only (A) and (B)
c. Only (B) and (C)
d. Only (A) and (D)

34. Statements:
Some cars are jeeps. All the boxes are jeeps. All the pens are cars.
Conclusions:
A. Some cars are boxes.
B. No pen is jeep.
C. Some boxes are cars.
Options:
a. None of three
b. Only (A) and (B)
c. Only (A) and (C)
d. Only (B) and (C)

solutions to syllogidms
Syllogisms Solutions
The answers for the Syllogism Questions are given along with their respective ven
diagrams. The solutions are easy to understand and are sure to effectuate the CAT
preparation.
1. (B). Only (B) conclusion follows
Solution:

2. (D). Neither (A) nor (B) follows


Solution:

3. (B). Only (B) conclusion follows


Solution:
4. (E). Both (A) and (B) follow.Solutions:

5. (E). Both (A) and (B) follow.


Solutions:
6. (B). Only (B) conclusion follows
Solutions:

7. (E). Both (A) and (B) follow.


Solutions:
8.(D). Neither (A) nor (B) follows

9. (B). Only (B) conclusion follows


Solutions:

10.(B). Only (B) conclusion follows


Solutions:
11.(D). Neither (A) nor (B) follows.

12.(E). Both (A) and (B) follow.


Solutions:

13.(D). Neither (A) nor (B) follows


Solutions:

14.(A). Only (A) conclusion follows


Solutions:
15.(D). Neither (A) nor (B) follows.

16.(D). Only (C)


Solutions:

17.(A). Only (A) and (B)


Solution:

18.(C). Only (C)


Solution:
19.(C). Only (B) and (D)
Solution:

20.(D). Only (D).21.(E). None of the four.


Solution:

22.(D). None of the four.


Solution:
23.(C). Only (A), (C) and (D)
Solution:

24.(A). Only (C).


Solution:

25.(A). Only (C) and (D).


Solution:
26.(B). Only (A).
Solution:

27.(D). Only (A) and (D).


Solution:
28.(B). Only (A) and (D).
Solution:

29.(B). Only (C).


Solution:
30.(A). Only (A).
Solution:

31.(E). Only (C).

32.(E). None of the four.


Solution:
33.(A). Only (C) and (D).
Solution:

34.(A). None of three.


Solution:
Test Miscellaneous Arithmetic- Questions
1)A bottle contains 100% phenyl. 1/3 of it is replaced with water.The operation is
repeated 4 times.What is the ratio of phenyl : water after the last operation?

a) 16:81

b) 1:5

c) 14:82

d)none of these

2) A mixture of 125 litres of milk and water contains 20% water.What amount of
water needs to be added to this milk-water mixture in order to increase the
percentage of water to 25% of the new mixture?

a) 7

b) 5.66

c) 8.33

d) none of these

3)There is a bottle of Coke. 25% of it is replaced by Pepsi. Again 25% is replaced


by Pepsi and finally, another 25% is replaced by Pepsi.What is the approximate
final percentage of Coke in the bottle?

a) 56

b) 52

c) 66

d) 42

4)Two casks contain mixtures of petrol (P) and kerosene (K). In the first cask
P:K=7:3 and in the second cask P:K= 3:1.In what ratio should the mixtures from
the two casks be taken to give a mixture in which P:K is 11:4?

a) 2:3

b) 2:2

c) 1:2

d) 1:3
5)A rice trader sells one type of rice at Rs 2.70/ kg and loses 10%. He sells
another type of rice at 4.5/kg and gains 12 ½ %. His objective is to mix these two
types and sell the mixture at 3.95/kg and get a profit of 25%.What should be the
ratio of quantities of rice mixed?

a) 23:6

b) 84:16

c) 6:1

d) none of these

6) A student finds the average of 10 positive integers. Each integer contains two
digits.By mistake, the boy interchanges the digits of one number say ba for ab.
Due to this, the average becomes 1.8 less than the previous one.What was the
difference between the two digits a and b?

a) 2

b)4

c)6

d)none of these

7) C and D are the children of A&B, who are married to each other. D is 5 years
younger than C.Five years ago, A was twice as old as C. The current age of B is the
average of A and C’s ages.If the average age of all of them is 34 years, find the
current age of D?

a) 18

b) 21

c) 22

d) 27

8) Arun invests in bonds worth Rs. 40000 and he receives a 15% interest per
annum.How much money should he invest in bonds that give 10% per annum so
that he gains overall 12% per annum?

a) 50000

b) 65000

c) 60000
d) 62500

9) The average weight of a group of 8 girls is 50 kg. If 2 girls R and S replace P and
Q, the new average weight becomes 48 kg.The weight of P= Weight of Q and the
weight of R= Weight of S. Another girl T, is included in the group and the new
average weight becomes 48 kg.Weight of T= Weight of R. Find the weight of P?

a) 48

b) 52

c) 46

d) none

10) Disha wants to paint her room a unique brown. For that she uses a certain
violet paint which contains 30 % blue pigment and 70 % red pigment by weight.
She mixes that violet paint with a certain green paint which contains 50 % blue
pigment and 50 % yellow pigment.When these paints are mixed to produce the
brown paint which she uses, it contains 40 % blue pigment.If the brown paint
weighs 10 grams, then the red pigment contributes how many grams of that
weight?

a) 2.8 gms

b) 3.5 gms

c) 4gms

d) 5.3 gms

11) A set S of real values consists of the following 5 elements, {4, 8, 12, 16, x} in
any order.For how many values of x does the mean of set S equal the median of set
S?

a) 0

b) 1

c) 2

d) 3

12) The average age of ‘n’ persons is 55. Two persons aged 41 and 47 respectively
leave the group and three new persons aged between 60 and 65 join the group.If
the average age of the group goes up by one year and if the initial number of
people in the group is a multiple of 7, what is the number of persons now in the
group?
a) 42

b) 43

c) 44

d) 45

13) Manju, who works at Ganesh Fruit Juice Centre, prepares 51 glasses of
milkshake in 4 min, 18 sec in his first shift.After having a sandwich, he prepares
73 glasses in 7 min 13 sec. After another sandwich, he prepares 112 glasses in 12
min, 24 sec.The container in which the milk shake is prepared contains a
maximum of 9 glasses of a drink at a time.Find his approximate average time of
preparation of one container of the drink, if he starts with a fresh container each
time.

a) 48s

b) 50s

c) 40s

d) 51s

14) There is a bag with a mixture of sugar, rice and wheat grains. The quantity of
rice is 4/5th that of wheat and the quantity of sugar is 3/4th that of rice. In 12 kg
of the mixture how much wheat and sugar will be there?

a) 7 kg

b) 10 kg

c) 8 kg

d) none of these

15) At a gold shop’s annual promotion, gold coins are distributed with each
purchase. The higher the purchase, the higher the value of the gold coin.The value
of the gold coin is directly proportional to the thickness & the square of the
diameter. Two gold coins are given to a customer. The value of coin 1: coin 2 =4:1.
The diameter of the coins are in a ratio 4:3.Find the ratio of the thickness of the
coins?

a) 9:6

b) 11:7

c) 9:4
d) 12:5

16) There are 5 friends in a class who get marks in the ratio 3:4:5:6:7. the
maximum marks are the same in each case.The total marks obtained by the 5 are
3/5th the maximum obtainable marks.How many friends got more than 70% of
the total marks?

a) 1

b) 2

c) 3

d) 4

17) The charges for a graphics designer are partially fixed and partially variable
with the number of hours.The charge is Rs 550 for 4.5 hours and Rs 300 for 2
hours. Find the charges of 1 hour?

a) 100

b) 200

c) 300

d) 400

Questions 18-19

In Little flowers school, for a school drill, the students are divided into 2 groups of
lilies and daisies.The ratio of lilies: daisies = 8:3. the ratio of boys: girls is 7:4.
60% of the Daisy group is boys.

18) What is the ratio of the number of girls in the lily group and the number of
boys in the daisy group?

a) 20:13

b) 11:20

c) 14:9

d) cannot be determined

19) What is the difference in the number of boys who are in the lily group and the
number of girls in the daisy group, given that there are 48 girls in the daisy
group?

a) 200
b) 240

c) 160

d) none of these

20) Till the age of 21, Arun grows such that his height varies as the square root of
his age. When Arun is 9 years old, his height is 4 feet. What is his height at 16
years?

a) 5 feet

b) 5 feet 4 inches

c) 6 feet

d) 6 feet 2 inches

21) The height of an apple tree is given by the sum of two terms. The first term is
directly proportional to the weight of the tree and the second term is directly
proportional to the square of the weight of the tree. The height of the tree was 6 m
and 8 m, when its weight was 40 kg and 50 kg respectively.Find the approximate
weight of the tree when its height was 5 m.

a) 39 kg

b) 25 kg

c) 32 kg

d) 35 kg

22) A picnic invites two kinds of charges: bus fare, which is independent of the
number of people attending the picnic and buffet lunch, which increases directly
with an increase in the number of people.The charges are calculated to be Rs. 165
per head when there are 200 invitees and Rs. 170 per head when there are 150
invitees.What would be the charges per head when there are 100 invitees?

a) Rs.175

b) Rs.180

c) Rs.185

d) Rs.190

23) Praveen buys two cans of milk from a manufacturer. The two cans are diluted
to the same extent. To can 1 Praveen replaces 10 litres of the solution with pure
milk and the concentration of milk becomes twice of what it was.To can 2 he
replaces 20 litres with pure milk, what is the ratio of concentration of can 1
initially to that in can 2 finally.

a) 1:1.5

b) 1:2

c) 1:3

d) cannot be determined

24) The receipts on railway travel vary as the excess of speed of the train over 30
kmph. The expenses vary as the square of that excess.What is the speed at which
the profits will be greatest if at 60 kmph is the expenses are just covered?

a) 40

b) 35

c) 45

d) None of these

25) There are 3 types of Coffee powder available at La Café. They are graded
based on the chicory content.The three grades of coffee contain 1% chicory, 2%
chicory and 3% chicory in the coffee blend. If p kgs of the 1% grade and q kgs of
the 2% grade and r kgs of the 3% grade are mixed to give p+q+r kgs of 1.5%
grade, what is p in terms of q and r?

a) q+3r

b) (q+r)/4

c) 2q + 3r

d) 4.5q+3r

solutions

Miscellaneous Arithmetic- Solutions


Check the detailed solutions for Miscellaneous Arithmetic Questions here.
1) option (d)

(Liquid A left after nth operation)/(Initial quantity of A in the vessel) = (a-b)n/an


Amount of phenyl remaining = (1-1/3)4 =16/81

Therefore, ratio or phenyl:water = 16:65. (81-16=65)

2) option (c)

We need to find out how much of a solution of 100% water needs to be added to a
solution containing 20% water to attain a dilution of 25%.

This can be found out as follows

Ratio = 75:5 = 15:1

That is for 15 parts of a 20% water solution, one part of 100% water solution needs to
be added.

Therefore, for a solution of 125 litres, 125/15= 8.33 litres need to be added.

3) option (d)

The answer is based on (100-25)3= 753. This translates to 42.18%

4) option (b)

Using alligation, we can directly solve this question Take either petrol or kerosene.

Petrol in cask 1 = 7/10 Petrol in cask 2 = 3/4 Petrol in mixture = 11/15

Ratio to take the mixture = 1/60:1/30= 1:2

5) option (b)
This question should be solved using alligation.

For alligation; always consider the cost price only.

Selling Price (SP) Cost Price(CP)

SP of rice 1 = 2.70/kg CP = 2.7/0.9 = 3/kg

SP of rice 2= 4.5/kg CP= 4.5/1.125 = 4/kg

SP of mixture = 3.95/kg CP= 3.95/1.25 = 3.16/kg

Required Ratio = 84:16

6) option (a)

We can infer that the difference between ab and ba is 18. Now all numbers of type 13,
24,35.

I.e. the difference is 2 will give a difference of 18 when reversed. Therefore the
difference of a and b will always be 2.

7) Option (c)

Let the present age of each of them be the letters themselves.

Present age of D= C-5

Present age of A will be 2(C-5)+5 =2C-5 and the present age of B will be (C+2C-5)/2 =
(3C-5)/2

Average age of members =34

C+C-5+2C-5+ (3C-5)/2=34×4

4C+(3C-5)/2= 146 11C= 297 C=27 D= 27-5=22

8) Option (c)

Using alligation
He should invest in the ratio 2:3.

He should invest 60000

9) option (d)

8 x 50 +R+S-P-Q= 48×8

R+S-P-Q=-16 P+Q-R-S= 16

R=S and P=Q P-R=8

One more person is included and the weight = 48 kg

Let the weight be a (48×8 +a)/9= 48

a=48 kg= weight of R weight of P= 48+8= 56 kg

10) Option (b)

Use Alligation:

Considering only the blue pigment in all the paints

Thus, 5 gms of each paint is used. Weight of red paint = 7/10 x 5 =3.5 gms. Option (b)

11) option (d)

To solve this problem quickly, you can come up with likely values for x that would make
the mean equal to the median.

Three values that would make the set symmetrical are 0, 10, and 20:

{0, 4, 8, 12, 16} {4, 8, 10, 12, 16}


{4, 8, 12, 16, 20}

To verify if there are any other possibilities for x, we need to evaluate each range of “x”

(1) If x is less than or equal to 8, then the median is equal to 8. We now set the mean
equal to the median: (40 + x)/5= 8 40 + x = 40 x = 0

(2) If x is between 8 and 12, then the median is equal to x. (40 + x)/5= x 40 + x = 5x 40 =
4x x = 10

(3) If x is greater than 12, then the median is equal to 12. (40 + x)/5= 12 40 + x = 60 x =
20

12) Option (b)

Let x, y, z be the ages of 3 persons which lies between 60 and 65 The equation governing
the problem (55n-41-47+(x+y+z))/(n+1)=56

Also given n is a multiple of 7 55n – 88 +k = 56n + 56 where k =x+y+z, n+ 144=k

Now k ranges between 3×61 and 3×64 183 ≤n +44 ≤ 192 39 ≤n ≤4 Thus n =42

The latest number n + 1 =42+1=43

Shortcut!

Go from answer options. The initial number of people in the group is a multiple of 7.

The answer options give the number of initial people + 1. Thus the correct answer
option is option (b)

13) option (d)

In the first shift, he uses the container 6 times. 45 glasses in 5 rounds and the remaining
6 glasses in the 6th round.

Similarly, he prepared 73 glasses in 9 rounds, and 112 glasses in 13 rounds. Hence, the
required average

= total time/ total number of rounds = (4 min⁡18 s + 7 min➜13 s + 12 min➜24 s)/(


6+9+13)=(23 min+55s)/28 = 51 seconds

14) option (c)


If we take the amount of sugar as 3x, then rice = 4x and Wheat = 5x

(ratio = 3:4:5; as rice= 4/5 wheat and sugar = 3/4 rice)

12x= 12➜ x=1 Wheat = 5 kg and Sugar = 3 kg.

Total = 8 kg

15) option (c)

V➜ Value d➜ diameter t➜ thickness

given that ➜ V α d2t ratio of diameters = 4:3 and

thickness = t1:t2

V1/V2 =(16 x t1)/(9 x t2 ) ➜ 4= (16t1)/(9t2 ) ➜ t1:t2= 9:4

16) Option (b)

Lets assume that the maximum marks are 100.

Total marks available =500 Aggregate = 3/5 x 500 = 300

Which is divided in the ratio 3:4:5:6:7 Marks are 36, 48,60, 72 and 84.

Only 2 students get more than 70%

17) Option (b)

The total charge = Fixed component +Variable component

Z= F+kx

Where F and x are fixed 550 =F +k(4.5) 300=F + k(2)

Solving simultaneously, F= 100 and k= 100

Charges for 1 hour = 100 +1(100) =200

Questions 18-19

Solution:

Since 8+3=7+4=11, the safest assumption is that there are 110 students in total.
Then, the table will look like this

<tdstyle=”border: 1px solid #E0DDDD !important;”>80

BOYS GIRLS

LILIES 52 28 80
DAISIES 18 12 30
70 40 110
Question 18 ➜28:18 = 14:9

Question 19➜ 1.2x=48 => x= 40

So the number of boys in lilies will be 5.2 × 40 = 208

So the difference will be 208-48 = 160

20) option (b)

Relationship given:- H α √A

H=K√A

4= K√9 ➜ K=4/3

Height of Arun at 16 years➜ H=4/3× 4 =16/3= 5 feet 4 inches

21) option (d)

Let the weight and height of the tree be ‘w’ and ‘h’ respectively.

Thus, h = aw + bw2 where a and b are constants.

Here,

6 = 40a + 402×b 8 = 50a + 502×b

Hence, a = 11/100 and b = 1/1000 If h = 5, 5 = 11/100×w + 1/1000×w2 Solving, w = 35


approximately.

22) option (b)

Let the bus fare = Rs. x and buffet lunch per head = Rs. y

x + 200y = 165 × 200 …(i)


x + 150y = 170 × 150 …(ii)

Then, x + 100y = 2 × (ii) – (i) = 51000 – 33000 = 18000

Therefore, Cost per head = Rs. 180

23) Option (c)

If the addition of 10 litres of pure milk makes the concentration twice of what it was
before, then the amount added will be the amount of milk present initially.

Replacement of 20 litres, will be the replacement with two 10 litres, making the
concentration thrice of what it was initially.

Therefore, the ratio = 1:3

24) Option (c)

Let the excess of speed over 30 kmph=S

Receipts= R and Expenses=E

Then

R=K1 S And E= K2S2

Also, at 60kmph, R=E Thus, K1=30 K2

We need to maximize R-E= K1S- K2S2=SK2(30-S) S+30-S,

the sum is constant, the product will be maximum when they are equal Thus S=30-S =>
S=15.

Speed = 30+15= 45

25) option (a)

Put r=0, then if 1 kg of 1% grade is mixed with 1 kg of 2% grade,

we get 2 kg of 1.5%grade Hence p=1, q=1 and r=0.

Substitute in answer options to get answer as option (a)


https://byjus.com/free-cat-prep/test-miscellaneous-arithmetic

Time Speed Distance: Practice Questions


1. Amir travels half of his journey by Bus at a Speed of 200/9m/s and
half of his journey by Metro at 120km/h. Calculate his average Speed
over the entire journey.
a) 86 km/h
b) 75 km/h
c) 90 km/h
d) 96 km/h

2. Ram by bus takes double the Time taken by train to travel from
Bangalore to Chennai. What is the Speed of the train if the Speed of
bus is 40 km/hr.
a) 40 kmph
b) 60 kmph
c) 80 kmph
d) 30 kmph

3. Vivek and Bharath go home daily after Office by an Office Cab which
has a Speed of 40 kmph. Vivek takes 20% more Time than Bharath to
reach his home. If Bharath’s house is at a Distance of 30 km from the
office, then calculate the Distance of Vivek’s house from the office.
a) 36 km
b) 40 km
c) 45 km
d) 42 km

4. One day Prakash started late for office by 1 hour, so he increased his
normal Speed by 5 km/hr so that he reaches on Time. Find the normal
Time taken to reach his office if his office is at a Distance of 60 km from
his house
a) 3 hrs
b) 5 hrs
c) 6 hrs
d) 4 hrs
5. If Sajesh increases his Speed from 12 km/hr to 15 km/hr while coming
from Office to home, he reaches home one hour early. Determine the
Distance between his home and the office.
a) 40 kms
b) 50 kms
c) 60 kms
d) 70 kms

Questions 6-7:
A starts from home for his office. He travels downhill, then on flatground and
then uphill to reach his office.
It takes him 3 hrs to reach the office. On the way back home A takes 3 hrs 10
min to reach home along the same route.
The Speeds downhill is 60 km/hr, on flat ground is 48 km/hr and uphill is 40
km/hr.

6) What is the Distance between A’s home and his office?


a) 144 km
b) 148 km
c) 154 km
d) 100 km

7) By what Distance should his office be shifted so that the Time taken
to go to the office is same as Time taken to reach home from the office?
a) 20 km
b) 30 km
c) 40 km
d) 15 km
Directions for questions 8-9:
The figure represents a square track in a racing game with O as the central point
.all the concentric tracks,
i.e. ABCD, EFGH, IJKL are all squares. Cross Lines like RNJFB and DHLPT pass
through the centre and run across the square tracks.
The Distances of points A,B,C,D is 2x from O and that of E,F,G,H is 4x from O,
that of I,J,K,L is 8n from O and so it continues.
There are 3 players P1,P2 and P3 positioned at Q,N and R respectively.
The aim is to reach the finish flag placed at B first. They can move only along
the tracks and the Cross lines
8) If both P1 and P2 start running simultaneously, then the ratio of the Time
taken for them to reach the point B is?
a) 1:2
b) 3:4
c) 4:1
d)√2:1

9) Given that P1,P2 P3 start running towards the finishing post “B” at
the same Time. What is the ratio of their Speeds if they all reach the
finishing post simultaneously?
a) 1:1:2
b) 2:3:5
c) 5:6:8
d) 3:7:4

10) Amit and Bala leave at 8 am everyday to meet each other at point X
after 2 hours. On one day, Amit walks at 5/6th of the usual Speed while
Bala starts one hour late.Bala thus increases his Speed by 25%. Now
Amit takes ½ hour more than usual to meet Bala and they meet ½ km
away from the point X.
Find out the Speeds of A and B and the total Distance travelled by
them?
a) 5 kmph, 5 kmph, 20 km
b) 6 kmph, 4 kmph, 22 km
c) 6 kmph, 4 kmph, 20 km
d) 4 kmph, 6 kmph, 20 km

11)A 50m long platoon is marching ahead. The last person in the platoon
wants to give a letter to the first person leading the platoon.So while the
platoon is marching he runs ahead, reaches the first person and hands
over the letter to him and without stopping he runs and comes back to his
original position.In the mean Time the whole platoon has moved ahead
by 50m. How much Distance (approximately) did the last person cover in
that Time.Assuming that he ran the whole Distance with uniform Speed.
a) 120m
b) 100m
c) 102m
d) 97m

12) Amar, Akbar and Anthony ran on a racetrack, with Amar finishing
160 m ahead of Akbar and 400 m ahead of Anthony.Akbar finished the
race 300 m ahead of Anthony. The three of them ran the entire Distance
with their respective constant Speeds. What was the length of the
racetrack?.
a) 600m
b) 800m
c) 1000m
d) 500m

13) A and B are moving in a circular track in the same direction. They start
simultaneously in a race which requires them to cover 15
rounds.Whenever A &B meet, it was found that the ratio of the number of
rounds covered by them till then is 3:1.The Time taken by B to complete
the race if they meet every 5 minutes is?
a) 75 mins
b) 100 min
c) 150 min
d) 50 min

14) A and B drive separately to an ice-cream parlour. A’s average driving


Speed is greater than B’s average driving Speed by 1/3rd of B’s driving
Speed, and A drives twice as many kilometers as B.What is the ratio of the
number of hours A spends driving to the parlour to the number of hours
B spends driving to the parlour?
a) 2:3
b) 4:3
c) 3:2
d) 8:3

15) Ahmed and Sahil set out together on bicycle traveling at 15 and 12
kilometers per hour, respectively.After 40 minutes, Ahmed stops to fix a
flat tire. If it takes Ahmed one hour to fix the flat tire and Sahil continues
to ride during this Time,how many hours will it take Ahmed to catch up to
Sahil assuming he resumes his ride at 15 kilometers per hour? (consider
Ahmed’s deceleration /acceleration before/after the flat to be negligible)
a) 4.5
b) 3 1/3
c) 3.5
d) 4

16) Ajay and Arun start running simultaneously from the diametrically
opposite ends of a circular track towards each other at 15km/h and
25km/h respectively. After every 10 minutes their Speed reduces to half
of their current Speeds. If the length of the circular track is 1500 m,how
many Times will Ajay and Arun meet on the track?
a) 6
b) 9
c) 11
d) 7

17) Katrina walks down an up-escalator and counts 150 steps. Priyanka
walks up the same escalator and counts 75 steps. Katrina takes three
times as many steps in a given Time as Priyanka.How many steps are
visible on the escalator?
(a) 105
(b) 150
(c) 135
(d) 120

18) Two guys Abhinav and Bineesh are walking down an escalator in the
direction of the motion of the escalator.A takes three steps in the same
Time when B takes two steps. When A covers 90 steps he gets out of the
escalator while B takes 80 steps to get out of the escalator. If they start
from opposite ends using the same escalator, find the difference in steps
covered by them when they meet.
a) 48
b) 20
c) 30
d) 24

19) Two ships take different routes to reach the “spice capital of the
world”. The Distances traveled by the two ships are in the ratio of 3:2. Ship
1 moves at 40kmph and ship 2 travel at 60kmph.If the two ships arrived
at their destination with a Time gap of 1 hour, what is the Distance
traveled by both the ships together?
a) 72km
b) 120km
c) 150km
d) none

20) The ‘moving walkway’ in an uptown mall in Paris is a 300-metre long


walkway consisting of a conveyor belt that moves continuously at 3
meters per second. When Ishan steps on the walkway, a group of
teenagers that are also on the walkway stands 120 meters in front of
him.He walks toward the group at a combined rate (including both
walkway and foot Speed) of 6 meters per second relative to the
ground.Once Ishan reaches the group of teenagers, he stops walking and
stands with them until the walkway ends.What is Ishan’s average rate of
movement for his trip along the moving walkway?
a) 2 m/s
b) 3 m/s
c) 3.5 m/s
d) 5 m/s

21) Peter, Sana, and Gavin are visiting the Adams family who are staying
200km away. Each of their walking Speeds is 10 km/h. Initially, Peter and
Gavin travel in a car at the rate of 50kmph and Sana walks the
Distance.After a while, Gavin gets off the car as he feels nauseated and
walks the rest of the Distance to the house. Peter goes back in the car to
fetch Sana and they all reach the house at the same Time.What was the
entire Time involved in traveling?
a) 10 hours
b) 7 hours
c) 8 ½ hours
d) 8 hours

22) Anu and Varsha are running on a circular track at a rate of 44 m/min
and 22 m/min respectively. They start on the same point and run in
opposite directions. The diameter of the track is 140 m. When they both
meet for the 12th Time, what is the Distance that Anu would have covered
over Varsha?
a) 1500
b) 1825
c) 1760
d) none of these
23) Consider a circular track of circumference= 1400 m. There are 2 bikes
which start from point A and move in the opposite direction. Once they
meet, they start moving in the opposite directions. The one that moves in
the anticlockwise direction to the other one has a Speed in the ratio of
36:48 kmph.Find the Distance of the two bikes from A when they meet for
the 15th Time?
a) 1200
b) 250
c) 350
d) none of these

24) Afsan is deciding which car to rent for a day for a class trip, from
among anPinnova and a Bilto. The rate/km is in a ratio of 3:2, the seating
capacity is in a ratio of 5:2. The Speeds are in the ratio of 7:4.Find out the
ratio of the maximum cost incurred that day for the two car types, given
that there is no wastage of capacity or Time?
a) 60:28
b) 56:30
c) 105:16
d) 140:12

25) Amit and Aman have to travel from Delhi to Jaipur in their respective cars.
Amit is driving at 60 kmph while Aman is driving at 90 kmph. Find the Time taken
by Aman to reach Jaipur if Amit takes 9 hrs.
a) 6 hours
b) 4 hours
c) 2 hours
d) none of these

26) Ram and Shyam are standing at two ends of a room with a width of 30 m.
They start walking towards each other along the width of the room with a Speed
of 2 m/s and 1 m/s respectively. Find the total Distance traveled by Ram when
he meets Shyam for the third Time.
a) 60 m
b) 100 m
c) 200 m
d) none of these
27) Two trains A and B leave stations P and Q simultaneously and travel
towards Q and P respectively on the same route. After meeting en route,
A takes one hour to reach Q and B takes 4 hours to reach P.How long did
A take to cover the entire Distance?
a) 6 hours
b) 4 hours
c) 2 hours
d) none of these

solutions to time and distance

TIME SPEED DISTANCE – SOLUTIONS


1) Option (d)
As the Distance covered is same with both the Speeds, the average Speed will be the
harmonic mean of the individual Speed.
200/9m/s = (200*3600)/((9*1000) ) = 80 km/h
So, average Speed = (2*a*b)/(a + b) = (2 * 80 * 120)/(80 + 120) = 96 km/h.
2) Option (c)
Distance = Speed × Time
As Distance covered in both the cases is constant, Speed will be inversely proportional
to time.
So, if Time by bus: Time by train = 2: 1, Speed of bus: Speed of train = 1: 2
Speed of bus is 40 km/hr, so Speed of train is 80 km/hr.
3) Option (a)
Distance = Speed x Time
In both the cases, Speed is constant so Distance will be proportional to the time.
Vivek takes 20% more than Bharath. So, assume that Bharath takes 100 minutes then
Vivek will take 20% more i.e. 20 minutes more, so 120 minutes. So, the ratio of Time
taken by Vivek and Bharath = 120/100= 6/5. Distance is proportional to the Time taken,
so ratio of Distance of Vivek’s House and Bharath’s House = 6/5, Bharath’s house is at a
Distance of 30 km, so Vivek’s house will be at the Distance of (6/5)*30 = 36 km.
4) Option (d)
Method 1:
Suppose the normal Time taken is ‘t’ then when he is late by 1 hour, Time taken by him
= ‘t – 1’.
Normal Speed = 60/t ,Speed when he wants to reach in t – 1 Time = 60/(t – 1)(60/(t-
1))– (60/t) = 5? t2 – t – 12 = 0 ? (t – 4) (t + 3) = 0 ? t = 4 and t = – 3 (-3 is not possible)So,
normal Time taken = 4 hours. Hence option (d).
Method 2: Reverse Gear Approach
Take a middle answer option, say 5 hours Normal Speed = 60/5 = 12 kmph Speed
increased to 12+5=17 kmph. 60/17 = 3.53 hours,
the difference is more than 1 hour. This is not the answer. We can also eliminate 6
hours, as that will result in a higher Time difference.
Answer is either 3 or 4 hours(answer is close to 5) Trying for 4 hours, Normal Speed=
60/4 = 15 km/h
Speed increased to 20 km/h 60/20=3. 4-3 =1. This is the answer.
5) Option (c)
Sajesh increases his Speed from 12 km/hr to 15 km/hr, so increase in Speed = 3
km/hour 1/4. In both the cases, Distance covered remains the same. So, applying
constant product rule: His Speed increases by 1/4 so Time will decrease by 1/5.
Here Time is decreasing by 1 hour and 1 hour is 1/5 of the actual Time taken. So, actual
Time taken = 5 hours. So, Distance between Office and House = 5 * 12 = 60 kms. Hence
option (c). Alternatively, you can also check the answer options.
(a) Distance = 40 kms, actual Speed = 12. Actual Time taken = 40/12. New Time taken =
40/15.
(40/12)– (40/15)≠ 1
(b) Distance = 50 km, actual Speed = 12. Actual Time taken = 50/12. New Time taken =
50/15. (50/12)– (50/15)≠1
(c) Distance = 60 kms, actual Speed = 12. Actual Time taken = 60/12= 5 hrs. New Time
taken = 60/15 = 4 hrs.
Difference = 1 hour.
6) Option (b)
Let x,y and z be the Distances uphill, on flat ground and downhill respectively. Then

7) Option (a)
Let AC be 3 km more than DE. Then
Thus, if the Distance is decreased by 20 km, the Time taken will be the same.
8) Option (d)
Their Speeds are not given. So the ratio of the Time taken cannot be determined.
9) Option (d)
There is no traveling Distance because they can travel anywhere in the path. So the ratio
of their Speeds cannot be determined i.e., for example, if player P2 will take the path N-
J-F-B, or N-d-a-B etc. can never be determined.
10)Option (c)
If you observe the options carefully, you can decide which answer option to start with.
Since the fraction 5/6 is involved. Start with an option which is a multiple of 6; option
(b) and option (c) Now one has 2 Distances 20 km and 22 km A’s original Speed =
6kmph. In 2 hours he will travel 12 km B’s original Speed= 4 kmph. In 2 hours he will
travel 8 km 12+8=20 and not 22. Here itself, you can mark the answer as option (c)
Verifying A’s new Speed= 5 kmph B’s new Speed = 5 kmph After 1 hour A travels 5 km,
B travels 0 After 2 hours A travels 10 km (total), B travels 5. Total =15 km. They thus
meet after 2.5 hours.
11) Option (a)
It is given that the platoon and the last person moved with uniform Speed. Also, they
both moved for the identical amount of Time. Hence, the ratio of the Distance they
covered – while person moving forward and backward – are equal. Let’s assume that
when the last person reached the first person, the platoon moved X meters forward.
Thus, while moving forward the last person moved (50+X) meters whereas the platoon
moved X meters. Similarly, while moving back the last person moved [50-(50-X)] = X
meters whereas the platoon moved (50-X) meters. Now, as the ratios are equal
(50+X)/X = X/(50-X) (50+X)*(50-X) = X*X Solving, X=35.355 meters Thus, total Distance
covered by the last person = (50+X) + X = 2*X + 50 = 2*(35.355) + 50 = 120.71 meters
=120 m (approximately).
12) Option (b)
Using the concept of ratios of Speed (Speed directly proportional to Distance), we can
easily arrive at the totalDistance. The images below give a picture of the Distances when
Amar and Akbar finish the race
Length of the race track = 400 +x The ratio of Akbar and Anthony’s Speed can be arrived
at as (240+x)/x = (400+x)/(100+x)
Now go from answer options, answer will be option (b) 800 m Verifying, x = 400
640/400 = 800/500. Thus, answer is option (b).
13) Option (c)
The ratio of Speeds of A and B= 3:1, This means that when A makes 1.5 rounds, B makes
0.5 round. For B to complete the race of 15 rounds, it would take him 15/0.5 x 5 =150
minutes.
14) Option (c)
Solve the question using assumption If B’s Speed is 3, then A’s Speed is 3+(3/3) = 4 If A
travels 12 km, B travels 6 km Ratio of Time taken = A:B = 3:2. Option (c).
15) Option (b)
In 2/3 of an hour A travels 10 km in 40 min B travels 8 km in 40 min After one hour, A
would have still traveled only 10 km and B would have traveled 20 km, their relative
Speed = 15-12= 3 km/hr A gains this 10 km in 10/3 =3.33 hours.
16) Option (b)
the total Distance traveled before they come to a halt can be calculated as
40(1+1/2+1/4+1/8…)/6 = 40((1/(1-1/2))/6)=13.33 km They cover a Distance of 750
m for the first time they meet, and subsequently cover a Distance of 1500 m each Time
they meet, hence if they meet 8 Times, then the Distance covered will be 1500 * 8 + 750
= 12750, which is lesser than 13,333.33. If they meet 10 Times then the Distance would
be 14250>13333.33. Hence they meet 9 Times.
17) Option (d)
Let T be Time Katrina takes to make 25 steps. Then Katrina takes 3T to make 75, and
Priyanka takes 2T to make 150. Suppose the escalator has N steps visible and moves n
steps in Time T. Then Priyanka covers N + 2n = 150, N – 3n = 75. Hence N = 120, n = 15
=> Choice (d) is the right answer.
18) Option (c)
Let the escalator moves x steps when A walks down 90 steps. Total number of steps on a
stationary escalator = x + 90. When A takes 90 steps, B should have taken 60 steps and
the escalator x steps. So when B takes 80 steps, the escalator should have taken 4/3*x
steps. So, 4/3*x + 80 = x + 90 = Total number of steps in the escalator when it is
stationary. So x = 30. Hence, total number of steps = 120. By the Time they meet,
together they will 120 steps in the ratio 3:2. i.e 72 and 48 steps. So, the difference is 24.
Option (d).
19) Option (b)
Time taken by Ship 1 = 3x/40 hr Time taken by Ship 2 = 2x/60 hr 3x/40–2x/60= 1
10x=240 x=24 Distances traveled are 48 & 72= 120.
20) Option (d)
Consider that Ishan’s journey will end when the group reaches the end of the walkway
(as long as he catches up with the crowd before the walkway ends). When he steps on
the walkway, the crowd is 180 metres from the end. The walkway travels this Distance
in (180/3) = 60 seconds, and Ishan’s average rate of movement is 300/60 = 5 metres
per second.
21) Option (d)
Let x be the number of hours Sana walks at 10kmph. x will be the no. of hours Peter and
Gavin traveled by car at 50kmph. Let x1 be the number of hours that Gavin walks till
Peter reaches Sana. Let x2 be the number of hours when Peter and Sana travel in the car
till the destination. The total Distance traversed by each of them can be evaluated as
follows
Peter = 50x-50*1+50*2=200——————(1)
Sana= 10x+10*1+50*2=200—————— (2)
Gavin = 50x+10*1+10*2=200—————–(3)
Solving we get t1= 2 hours, t=3 hours, t2=3 hours. Total = 8 hours.
22) Option (c)
This problem can be solved by noticing that the Speeds are in a ratio of 2:1. The
Distances covered will, therefore, be in a ratio of 2:1. When they meet for the 8th Time,
they would have covered the entire Distance of the track 8 Times = 8 (2pr) .Anu would
have covered 8 Times the Distance and Varsha 4 Times the Distance (Total= 12, ratio
2:1) Difference = 4(2pr) = 4x 2 x (22/7)x(140/2)= 1760.
23) Option (d)
1400 m track is divided into 7 parts of 200 m each. The Speeds of the two bikes are in a
ratio of 3:4, each time they meet, they will cover a Distance in the ratio of 3:4 They will
thus meet for the first Time at 3/7th the Distance which will happen at point 4 is 600 m.
The second time they will meet at the starting point. 3rd Time at point 4… and so on.so
the 15th Time, they meet at 600 m which is 800 m from A.
24) Option (c)
Cost incurred (maximum ) a Speed x capacity x cost/km Required ratio= (7x* 5x* 3x)/(
4x* 2x* 2x)=105/16.
25) option (a)
As the Distance covered is constant in both the cases, the Time taken will be inversely
proportional to the Speed. In the problem, Speed of Amit and Aman is in ratio 60: 90 or
2:3. So the ratio of the Time taken by Amit to that taken by Aman will be in the ratio 3:2.
So if Amit takes 9 hrs, Aman will take 6 hrs.
26) Option (b)
This is an instance of Case 1 as illustrated above. When Ram meets Shyam for the
third Time, they together would have covered a Distance of 5d, i.e 5x30m = 150 m.
Ratio of Speed of Ram and Shyam = 2:1, so the total Distance traveled by them will
also be in the ratio 2:1 as the Time taken is constant. So the Distance traveled by
Ram will be 2/3 x150= 100 m.
27) Option (d)
A takes 1 hour after meeting and B takes 4 hours after meeting. Hence Time traveled
before meeting = v1.4 = 2 hours Time taken by A= 2+1 = 3 hours

Venn Diagrams and Sets Questions


1. In a class,110 students like physics, 80 like chemistry, 120 like mathematics
and 70 like English. 20 students like only physics, 15 like only chemistry, 30
like only mathematics and 15 like only English.
What can be the maximum possible number of students in the class?
a) 155
b) 210
c) 380
d) 230
2. Of the 200 candidates who were interviewed for an admin position at Noesis,
100 had a two-wheeler, 70 had a credit card and 140 had a mobile phone. 40 of
them had both, a two-wheeler and a credit card, 30 had both, a credit card and
a mobile phone and 60 had both, a two wheeler and mobile phone and 10 had
all three. How many candidates had none of the three?
a) 0
b) 20
c) 10
d) 18
3. If one out of every four in 100 students had a car and three out of five
students had a bike and half the students had a cycle and not more than 20 out
of 100 had exactly two of the three above mentioned vehicles, then the
maximum number of students having all three modes of transport is?
a) 20
b) 25
c) 17
d) cannot be determined
4. At a cricket match, out of the 2 lakh spectators, 90% were fans of team A,
85% of Team B, 70% of Team C, 75% of Team D and 80% of Team E. What is
the minimum number of spectators who were fans of all five teams if it is given
that each of these spectators supports at least one team?
a) 5
b) 100
c) 1000
d) none of these
5. A group of 100 people plays carom, snooker and chess. 90 people play
chess, 80 people play snooker and 80 people play carom. Find the maximum
number of people who play all three games, if each person plays at least one
game.
a) 70
b) 75
c) 65
d) none of these
6. In a college, 35% of the students play at least two out of three games-
Cricket, Hockey and Football. Football is played by 45% of the students. 15%
of the people play both Cricket and Hockey, while 10% of the people play all
the three games. If 7% of the students do not play any game, what percentage
of the students play only Football?
a)
10%
b) 15%
c) 25%
d) 30%
Directions for questions 7-8:
In a group of 500 people, 350 are Engineers, 250 are MBAs. Determine:
7. How many are both Engineers and MBAs?
a) 100
b) 75
c) 50
d) 150
8. How many are either only MBA or only Engineer?
a) 250
b) 300
c) 350
d) 400
9. In a class, there are 200 students, at least 140 of students like Maths, at least
150 like Science and at least 160 like English. What is the minimum number of
students who like all three subjects?
a) 50
b) 83
c) 100
d) 150
Directions for questions 10-11:
In an examination, it was found that every student has failed in at least one
subject out of the three subjects: English, Maths and Science.
28 students failed in English, 30 students failed in Maths and 32 students
failed in Science. 6 students failed in English and Maths, 8 students failed in
Maths and Science and 10 students failed in English and Science. The number
of students who failed in only one subject is 54. Also, 20 students failed only
in Maths.
10. Determine the number of students who appeared in the examination.
a) 50
b) 20
c) 40
d) 70
11. Determine the number of students who failed in English and Science but
not Maths.
a) 4
b) 7
c) 5
d) 6
Directions for questions 12-14:
In a music school, three instruments are taught: Tabla, Violin and Guitar. Out
of 278 students in the school, 20 learn Tabla and Violin, 23 learn Violin and
Guitar and 21 learn Tabla and Guitar. 9 students learn all three instruments.
It is known that equal number of seats in all three instruments classes. (If a
student is learning Guitar as well Tabla, then he occupies two seats: one in
Tabla Class and one in Guitar Class)
12. Determine the number of students who have occupied only one seat.
a) 232
b) 200
c) 197
d) 234
13. Determine the number of students who have occupied seats in Violin or
Guitar class but not in Tabla Class.
a) 160
b) 153
c) 175
d) 167
14. Determine the number of students who have occupied seats in Tabla and
Violin Class but not in Guitar Class.
a) 9
b) 11
c) 13
d) 7
Directions for questions 15-17:
There are 70 workers in a company out of which 40 are men. Also,
i) 35 are married
ii) 35 are engineers
iii) 18 married workers are engineers of which 9 are men
iv) 12 men are engineers & not married
v) 12 men are married& not engineers
15. How many unmarried engineers are working in the company?
a) 16
b) 18
c) 15
d) 17
16. How many unmarried women work in the company?
a) 4
b) 5
c) 6
d) none of these
17. How many graduate men are not married?
a) 12
b) 14
c) 13
d) can’t be determined
Directions for questions 18-20:
In a class of 150 students, none of the students opted for only cricket. None of
the students opted for only Hockey and Football. 60 students opted for cricket
or hockey or both. Also, all 150 students opted for at least one of these 3
games
18. What is the total number of students who opted for only football?
a) 40
b) 80
c) 90
d) Cannot be determined
19. What is the total number of students who opted for hockey?
a) 100
b) 60
c) 50
d) cannot be determined
20. What is the number of people who opted for only cricket and only Football?
a) 15
b) 30
c) 90
d) cannot be determined
21. A group of 80 people play atleast one of the games- carrom, snooker and
TT. 40 play carrom, 50 play snooker and 35 play TT.
If 14 people play both Carrom and Snooker, 20 people play both Snooker and
TT and 12 people play both TT and carrom, find the ratio of the number of
people who play carrom only to the number who play only TT?
a) 17:4
b) 13:1
c) 15:4
d) 10:1
22. In an examination, 38% of students failed in Science and 33% failed in
Maths while 19% failed in both the subjects. If the number of students who
passed in only Science is 700, then determine the total number of students
who appeared in the examination.
a) 500
b) 350
c) 5000
d) 3500
23. In a class 40 people are interested in Music alone and 21 are interested in
Dance alone. If the number of students who are interested in neither are equal
to the number who like both, what is the likely strength of the class?
a) 65
b) 57
c) 70
d) 62
24. A,B,C and D are contesting for the district elections. Of the total
population, 50% like A, 25% like B, 30% like C. All of those who don’t like any
of these like D. All those who like B, like both A and C.All those who like C, like
A as well. Number of people who like C and not B is what percent of those who
like D?
a) 0
b) 10
c) 15
d) 25
25. In a study XYZ on the effect of sleep on call center workers, following were
the observations made on a certain number of volunteers in the study. 75% of
the volunteers receive fewer than 6 hours of sleep and report feeling tired
during their shifts. At the same time, 70% of the volunteers who receive 6 or
more hours of sleep report no feelings of tiredness. If 80% of the volunteers
receive fewer than 6 hours of sleep, what percent of the volunteers report no
feelings of tiredness during their shifts?
a) 24
b) 21
c) 42
d) 19

Sets Solutions
Here are the detailed explanations of the Sets Questions. The solutions are easily
understandable so that the candidates can easily learn the topic and ace their CAT exam.
Test Your Sets & Venn Diagrams Basics Skills-Solutions

1. Option (d)
S = 110+80+120+70 = 380; X needs to be maximized.
I = 80.
S = I + 2II + 3III + 4 IV= 380.
To maximize X one needs to minimize the overlapped regions.
Assume III= 0, IV = 0.
So that 2II = 380-80 = 300. II = 150. X max = 80 + 150 = 230.

2. Option (c)
Given that S= 310 (i.e.100+ 70+ 140).
II +3III= 40+30+60 = 130
III=10 =>II = 100
S-X=II+2III
X=310-120= 190 => No. of candidates having none of the three = 200-190=10.

3. Option (c)
S = 25+60+50=135
X=100 (given)
S-X= II+2III
35= II+2III
The least value II can take is 1, as the number of people has to be a whole
number
2III= 34 =>III(max) = 17.

4. Option (d)
In this case, the minima of all is that part of the venn diagram which has 5
overlapping regions. This can be found using a shortcut technique.
To find the minimum number of spectators who are fans of all 5 teams, take
the difference of each from X (i.e.the value of atleast 1), add it up and call the
sum Y; then take the difference of Y from 100 again to get the value Here
X=100,
Step 1- Taking the difference of each from X
100-90=10
100-85=15
100-70=30
100-75=25
100-80=20
Total(Y) ∑=100
Step 2- Take the difference of Y from 100, 100-100=0 Minimum number of
spectators who are fans of all 5 teams = 0.

5. Option (b)
S= 90+80+80=250
X= 100
Using the Maxima of all shortcut
(S-X)/(N-1) = 150/2= 75.

6. Option (b)
Using the venn diagram approach

II +III= 35,
a+b=20
Hence percentage playing only Football = 45-10-20 =15%.

Questions 7 & 8:
Soln:
n (E + M) = n(E) + n(M) – n(E ∩ M)
500 = 350 + 250 – n (E ∩ M)
n (E ∩ M) = 100

7. Option (a)
Since, 100 are both Engineers and MBAs.

8. Option (d)
People who are only Engineer = 250
People who are only MBA = 150
No. of people who are either only MBA or only Engineer = 250 + 150 = 400

9. Option (a)
X = I + II + III = 200
S = I + 2II + 3III = 140 + 150 + 160 = 450
S – X = II + 2III = 450 – 200 = 250
For III to be the minimum, II has to be the maximum. Now, II can take the
maximum value of 200.
So, minimum value of III = 250 – 200= 50.

Questions 10 & 11:

Consider Maths:
It is given that 20 students failed only in Maths.
So, x + z + y + 20 = 30 ➜x + y + z = 10
Also x + z = 8 y + z = 6 x + y + 2 z = 14 ➜ z = 14 – 10 = 4, then x = 4 and y = 2
So, redrawing the Venn diagram
10. Option (d)
Number of students who appeared in the examination = 18 + 20 + 16 + 6 + 4 +
2 + 4 = 70.

11. Option (d)


The number of students who failed in English and Science but not
Mathematics = 6.

Questions 12-14:

Number of students = 278 = x + y + z + 11 + 12 + 14 + 9


Let us consider the total number of seats: x + y + z + 2(11 + 12 + 14) +
(3×9) = [x + y + z + 11 + 12 + 14 + 9] + [11 + 12 + 14 + 9 + 9] = 278 + 55 = 333.
So, number of seats in each class = 333/3 = 111
x = 111 – (11 + 9 + 12) = 79
z = 111 – (12 + 9 + 14) = 76
y = 111 – (11 + 9 + 14) = 77

12. Option (a)


No. of students who have occupied only one seat = x + y + z = 79 + 76 + 77 =
232.

13. Option (d)


No. of students who have occupied seat in Violin Class or Guitar Class, but not
in Tabla Class = 77 + 14 + 76 = 167.

14. Option (b)


No. of students who have occupied a seat in Tabla and Violin Class, but not in
Guitar Class = 20 – 9 = 11.

Questions 15-17:
18 married workers are engineers out of which
9 are men ➜ s + q = 18,
s=9➜q=9
12 men are engineers ➜ r = 12.
12 men are married ➜ p = 12.

15. Option (d)


No. of unmarried engineer in the company = 5 + 12 = 17.

16. Option (b)


No. of unmarried women = 5 {unmarried women: not men + not married i.e.
z).

17. Option (a)


No. of engineer men not married = r = 12.

Questions 18-20:
From the data given in the question, one can fill up the Venn diagram as
follows

From the information given, a+b+c+d=60, b =0.

18. Option (c)


The number of people who opted for only football= 150-60=90.

19. Option (d)


The number of people who opted only for hockey cannot be determined (d).

20. Option (c)


The number of people who opted for only football and only cricket = 0+90=90.

21. Option (c)


x- number who play all 3 games
80= 125 –(14+12+20) +x
x= 1, S=125, X=80
S-X=45=II+2III II+III=46.
So III=1 The Venn diagram can be represented as

Answer = 15:4
22. Option (c)

So, the % of students failed in at least one subject = 19 + 19 + 14 = 52 %


So, the % of students who didn’t fail even in one subject = 100 – 52 = 48% %
of students failed in Science = 38%  % of students who passed in Science =
100 – 38 = 62% % of students failed in Maths = 33%  % of students who
passed in Mathematics = 100 – 33 = 67% Now, consider the % of students
who passed:

So, as we see from the Venn diagram, % of students passed in only Science =
14%. If the total no. of students who appeared = x, then .14x = 700  x = 5000.

23. Option (a)

be odd always .option c and d is out Hence x= 2. option a = 65 is possible.

24. Option (b)

From the Venn diagram given, it’s clear that no. of people who like C and not B
are 30 – 25 = 5. People who like D are 100 – 50 = 50. Therefore, 5/50× 100 =
10%.

25. Option (d)


Since the question is about percentages, assume a base value 100 (total
number of volunteers)
No. of volunteers receiving < 6 hours of sleep = 80
No. of volunteers who receive > 6 hours of sleep = 20 70 % of the above report
no feeling of tiredness =14
Number of volunteers with > 6 hours of sleep and report tired = 6
Volunteers who receive < 6 hours of sleep and report feeling tired =
75
Number of volunteers who receive < 6 hours of sleep and are not tired = 80-
75= 5
Total percentage of volunteers who report no feeling of tiredness during shifts
= 14+5 = 19%.
So, the total percentage of volunteers who report no feeling of tiredness
during shifts = 14+5 = 19%.

1) Find the value of ‘n’ if 12 x nP =nP .


6 4

a) 8
b) 11
c) 9
d) 7
2) Find the value of x such that: 7C -1 + 7C = 8C +2
x x x

a) 2
b) 3
c) 4
d) 5
3) Find the maximum value of 16C for any natural number x.
x

a) 6
b) 4
c) 7
d) 8
4) If nC = 5 and nP = 120, then determine the value of n, r.
r r

a) 5, 3
b) 4, 2
c) 7, 4
d) 5, 4
5) Determine the number of ways in which 5 prizes can be distributed among 4
students.
a) 45
b) 54
c) 20
d) 5!/4
6) How many different three-digit numbers can be formed with the digits 1, 2, 3,
4, 5 and 6.
So that none of the digits are repeated?
a) 120
b) 130
c) 150
d) 100
Directions for questions 7-8
7)How many 4 digit numbers greater than 5000 can be formed by using the digits
2, 3, 5, 6, 7 and 8 such that
None of the digits are repeated?
a) 216
b) 240
c) 360
d) 120
8) Digits can get repeated?
a) 765
b) 432
c) 312
d) 864
9) In how many ways can the letters of the word “CONVENIENCE” be arranged?
a) 11!
b) 11!/[(2!)(3!) ]
c)11!/[(2!)(3!)(3!)]
d) 11!/3!
10) In how many ways can the letters of the word “CONVENIENCE” be arranged so
that they begin with 2Ns and end with 2Es?
a) 7!
b) 11!/[(2!)(3!)(3!) ]
c)7!/3!
d) 7!/2!
11) A triangle ABC has 2 points marked on the side BC, 5 points marked on the
side CA and 3 points marked on side AB.
None of these marked points is coincident with the vertices of the triangle ABC.
All possible triangles are constructed taking any three of thesepoints and the points A,
B, C as the vertices.
How many new triangles have atleast one vertex common with the triangle ABC?
a) 256
b) 237
c) 207
d) 127
12) If all the 5 letter words that can be formed using the letters of the word
RASAM are arranged as in a dictionary, what will the rank of RASAM be?
a) 40
b) 35
c) 41
d) 42
13) Find the number of natural numbers which lie between 108 and 109 which
have products of their digits as 6?
a) 55
b)105
c)81
d) 92
14) In a board meeting, 8 delegates from number 1 to 8 are sitting around a
circular table.
Number 4 and 5 always sit together and number 1 always sits next to number 4. If
number 8 always sits exactly opposite number 3, how many can the seating be done?
a) 6!
b) 6C x 3
2

c) 4×3!
d) 4! x 2
15) A bag has 3 red balls, 2 yellow balls and 3 black balls. They are drawn one by
one and placed in a row.
Find the number of ways they can be arranged.
a) 280
b) 410
c) 560
d) 712
16) A basket has 5 oranges and 4 apples. In how many ways can you make a
selection if you have to take at least 1 orange and 1 apple?
a) 20
b) 22
c) 345
d) 465
17) In a college a committee of 7 people has to be selected from a group of 8
fourth year and 6 third year students. In how many ways can this committee be
selected if in the committee, majority of fourth year students is required?
a) 2320
b) 1960
c) 1530
d) 2416
Directions for questions 18-19
There are 5 boys and 4 girls. How many ways they can be arranged such that
18) No two girls are together
a) 3600
b) 4200
c) 2400
d) none of these
19) All girls are together
a) 5!x4!
b) 6!x4!
c) 4!x3!
d) 9!
20) How many different 7 digit numbers can be formed by using only 4,5,6 such
that each of the numbers has the digit 4 appearing twice?
a) 562
b) 672
c) 982
d) 1234
21) A grid is set up as shown using 5 horizontal and 6 vertical lines. What are the
number of ways one can go from point P to point Q walking along the grids but not
moving upwards or left or retracing a grid?

a) 21
b) 35
c) 56
d) None of these
22) 16 points are plotted on 3 parallel lines. What is the maximum number of
triangles that can be drawn with these points?
a) 416
b) 340
c) 520
d) none of these
23) An eight digit number divisible by 9 is to be formed by using 8 digits out of the
digits 0,1,2,3,4,5,6,7,8,9 without replacement.
The number of ways in which can be done is
a) 9!
b) 2(7!)
c) 4(7!)
d) (36)(7!)
24) How many 4-digit numbers that are divisible by 4 can be formed from the
digits 1,2,3, 4 and 6?
a) 36
b) 72
c) 24
d) None of these.
25) There are 7 couples in a dance party. How many dance pairs can be there such
that exactly 2 pairs are the original couples?
a) 765
b) 924
c) 1222
d) 904

Permutation and Combination


Solutions
Detailed solution of Permutaion and Combination

1)12x[n!/(n – 6)!]=n!/(n – 4)!

➜ 12(n – 4)! = (n – 6)! ➜ 12 (n – 4)! = (n – 6) (n – 5) (n – 4)!

➜ 12 = n2 – 11n + 30 ➜ n2 – 11n + 18 = 0

➜ (n – 9) (n – 2) = 0 n = 9, n = 2

Neglecting n = 2, n = 9.

Hence option (c)

2) We have nC + nC = n + 1C r-1 r r

7C + 7C = 8C ➜8C = 8C +2
x-1 x x x x

➜ Either x = x + 2 or x + x + 2 = 8But x ≠ x+2

➜ 2x + 2 = 8 ➜ x = 3

Hence option (b) Alternatively, go from answer options

3) Maximum value of 16C occurs when x = 16/2 = 8. x

Hence option (d)

4) (nP )/(nC )=120/5 = 24➜nC = nP /r!


r r r r

➜nPr/[n<subPr/r!] = 24➜r! =24

➜r=4

Given nP = 120 r

➜nP = 120 ➜n!/(n-4)!= 120


4

➜n(n-1)(n-2)(n-3) = 120 ➜ n = 5
Hence option (d)

5) This is a type of “Different to Different” question. 5 prizes can be distributed among 4


students in 45ways.

Hence option (a)

6) Suppose we start with the units place. Unit place can be filled by any of the 6 digits
given, so it can be filled in 6 ways. Now coming the tens place,

it can be filled by any of the six digits except the one which has been used at units place.
So, it can be filled in 5 ways.

Similarly the hundred’s place can be filled by any of the six digits except those two
which have been used at unit’s and tens’ place.

So, it can be filled in 4 ways

So, total numbers of numbers possible = 6 x5x4 = 120. Hence option (a)

Hence option (a)

Solutions for questions 7-8

7) When none of the digits are repeated:

The hundreds place can be filled by any of the digits: 2, 3, 5, 6, 7 or 8 except the one
which has already been used at the thousands place,

so it can be filled in 5 ways. Similarly tens place can be filled in 4 ways:

only those 4 numbers which have not been use either at hundred’s or thousand’s place.

Units place can be filled in only 3 ways.

So, total number of nos. Possible = 4x 5x4x3 = 240

Hence option (b)


8) When the repetition is allowed:

All the places: Hundreds, ten’s and units can be filled by any of the digits: 2, 3, 5, 6, 7 or 8
i.e. in 6 ways. Thousand’s place can be filled only in 4 ways.

Hence no. numbers possible = 4x6x6x6 = 864.

Hence option (d)

9) We need to arrange 2 Cs, O, 3Ns, V, 3Es and I. This is permutation with repetition. It
can be done in 11!/[(2!)(3!)(3!)] ways.

Hence option (c)

10) NN_ _ _ _ _ _ _EE The 7 places have to be filled by 2 Cs, O, N, V, E and I. This can be
done in 7!/2!

Hence option (d)

11) Total points including the vertices = 13 Out of these 13 points, 5 points on side AB (
including B and A) are collinear 7 points on side CA (including C and A) are collinear 4
points on side BC (including B and C) are collinear Thus, total triangles possible 13C – 3

5C – 7C – 4C = 237 Out of these,


3 3 3

we should exclude those triangles which do not share theVertices A or B or C Such


triangles = 13 – 3C – 5 – 2C – 7 – 2C – 0 = 109
3 3 3

We should also exclude the triangle ABC itself. Hence, the answer is 237 – 110 = 127.

Hence option (d)


12) First we fix A as the first letter No. of words possible with A as the first letter = 4! =
24

No. of words possible with M as the first letter= 4!/2! = 12

No. of words possible with RAA as the first 3 letters = 2

No. of words possible with RAM as the first 3 letters = 2

As of now, we have reached 24+12+2+2= 40

Next word will be RASAM

Rank= 41

Hence option (c)

13) 108<n<109 n is a 9 digit number Product of 6 is possible in the following cases

1) Exactly one digit is 6 and the remaining digits are 1 Number of possible numbers= 9
2) One digit is 2 and the other is 3, remaining are 1 = 9P = 72
2

Total possible numbers =72+9 = 81

Hence option (c)

14) Answer option c Lets fix number 8 at a slot at the table, automatically, number 3
also gets fixed.

Out of the remaining 6 delegates, 4 and 5 will be together and can be arranged in 2!
ways amongst each other.

Also for each of these ways, number 1 can sit next to 4 in only one way. Now 1,4,5 can be
placed only in 2 ways(See below).

This question now becomes the arrangement of the remaining 3 people x 4= 3! x 4.

Hence option (c)

15) There are a total of 8 balls with 3 of one kind, 2 of other kind and rest 3 of the same
kind.

These can be arranged in 8!/((3!)(2!)(3!)) = 560.

Hence option (c)


15) There are a total of 8 balls with 3 of one kind, 2 of other kind and rest 3 of the same
kind.

These can be arranged in 8!/((3!)(2!)(3!)) = 560.

Hence option (c)

16) At least 1 orange can be selected in 25 – 1 = 31 ways.At least 1 apple can be selected
in 24 – 1 = 15 ways.

Total no. of selections possible = 31 x 15 = 465.

Hence option (d)


17) Majority of fourth year means out of 7, minimum 4 should be from fourth year.
Considering different possibilities:

Hence option (d)

Solutions for questions 18-19

18) When no two girls are together, follow the following two steps

(i) Fix the positions of the 5 boys, with one space in between them.

This can be shown as _B1 _ B2 _ B3 _ B4 _ B5_ Thus the arrangement can be done in 5!
Ways

(ii) As you can see above, 6 places (gaps) are created when the boys are fixed.

The number of ways of selection of places for the 4 girls is 6C , and the number of ways
4

of arrangement is 4!.

Answer= 5!x6P 4

=43200
Hence option (d)

19) When all girls are together, follow the following three steps

(i) Take all the girls together as one unit. So totally there are 1+5 units (5 from the boys)

(ii) Arrange these 6 units. This can be done in 6! Ways.

(iii) Now arrange the girls among themselves in 4! Ways. Answer is option (b)

Total number of arrangements=6!x4!

Hence option (b)

20) 2 Slots for 4 from the available slots can be selected in 7C ways Remaining 5 slots
2

can be filled with any of 5 and 6 in 2.2.2.2.2 = 25 ways

Total ways = 7C x 25 = 672


2

Hence option (b)


21) There are 4 rows and 5 columns. Using the one-zero method, number of paths will
be 9C = 126 ways
4

Hence option (c)

22) The points should be distributed on the three lines as 6,5,5 .

6C (5C +5C ) + 5C (6+5) + 5C (6+5)=520


2 1 1 2 2

Alternatively, 16C -5C -5C -6C =520


3 3 3 3

Hence option (c)

23) From the digits 0 1 2 3 4 5 6 7 8 9

The sum is 45

Thus, every time we remove 2 digits which add up to 9, we will have a new set of 8
numbers which are divisible by 9

Case 1

Removing 0 and 9 Number of cases possible = 8!

Case 2

Removing

(1) 8 and 1

(2) 7 and 2

(3) 6 and 3

(4) 5 and 4

In each case, total number of cases = 8!-7! (we subtract 7! Because those are the cases
where 0 is the first digit) Hence,

total number of ways = 4(8!-7!)+8!= 36×7!.

Hence option (d)

24) To be divisible by 4, the last 2 digits should be divisible by 4 The combination of the
last two possible digits are 12 16 24 32 36 64 for each case,

the other 2 digits can be selected and arranged in 3P ways = 3!=6 ways
2

total possible cases= 6 x 6 =36

Hence option (a)


25) Number of ways of selecting 2 couples out of 7= 7C Derangement of 5 couples can
2

be done in 44 ways.

Total number of ways= 924

Hence option (b)

Probability Questions
Probability is one of the important topics of CAT examination, probability questions test
both Mathematical and Reasoning skill of a candidate, here we are providing some
questions which will help candidates to prepare for the examination.

Test Your Probability Skills – Questions


1)What is the probability of forming the word “ADDS” by picking 4
cards at random and kept in the same order out of 50 cards given with
10 cards having “A”, 10 having “S” and the remaining 30 having “D” on
it?
(a) 22/54
(b) 15/[(48-1)(48+1)x4]
(c) 1/502 (d) 10/47
(d) 10/47

2) A table is set up which can seat 5 people from 4 Adults and 6


children. Find the probability that the table seats at least 2 children?
(a) 31/42
(b) 124/126
(c) 41/42
(d) 3/26

3) For a drama, 3 students need to be selected out of 6 students A, B,


C, D, E and F. If A is already selected, what is the probability of selecting
C also?
(a) 0.5
(b) 0.2
(c) 0.3
(d) 0.4
4) If 30% of the visitors to an ice cream shop had chocolate ice-cream,
then what is the probability that 2 out of 3 people who entered the store
on Saturday at 5 pm will have chocolate ice cream?
(a) 0.16
(b) 0.15
(c) 0.19
(d) 0.28

5) A Number is selected at random from first thirty natural numbers.


What is the chance that it is a multiple of either 3 or 13?
(a) 17/30
(b) 2/5
(c) 11/30
(d) 4/15

6) What is the probability of getting at least one six in a single throw of


three unbiased dice?
(a) 1/6
(b) 125/216
(c) 1/36
(d) 91/216

7) What is the probability that a two digit number selected at random


will be a multiple of ‘3’ and not a multiple of ‘5’?
(a) 2/15
(b) 4/15
(c) 1/15
(d) 3/10

8) When two dice are thrown simultaneously, what is the probability


that the sum of the two numbers that turn up is less or equal to 10?
(a) 5/6
(b) 11/12
(c) 1/6
(d) 1/12
9) 4 out of 15 apples are rotten. They are taken out one by one at
random and examined. The ones which are examined are not replaced.
What is the probability that the 9th one examined is the last rotten one?
(a) 4C3× 11C5/15C8×1/7C1
(b)11C5×4C4 /15C9
(c) 4C3×11C5/15C9
(d) None of these

10) There are 22 red and 22 black balls in a box. Arun picks up three
balls at random. What is the probability that he has two balls of the
same color?
(a)2x 22C ×11C /44C
2 1 3

(b)22C ×22C /44C


2 1 (3

(c) 1
(d) None of these

11) A furniture shop has six identical steel cabinets of brand A and four
identical steel cabinets of brand B. Three customers buy one cabinet
each. Then the probability that two or more cabinets of brand A have
been sold
(a) 1/3
(b) 2/3
(c) 1/2
(d) 3/4

12) Arpit and Rita will play one game of Rock, Paper, Scissors. In this
game, each will select and show a hand sign for one of the three items.
Rock beats Scissors, Scissors beat Paper, and Paper beats Rock.
Assuming that both Arpit and Rita have an equal chance of choosing
any one of the hand signs, what is the probability that Arpit will win?
(a) 5/6
(b) 2/3
(c) 1/2
(d) 1/3

13) Two dice are thrown simultaneously. The probability of getting an


even number on both the dice is?
(a) 1/2
(b) 3/4
(c) 1/4
(d) 1/5

14) Parul and Vijay throw 3 dice in a single throw. It is known that Parul
throws a total of 16. Find Vijay’s probability of getting a higher value.
(a) 1/4
(b) 1/3
(c) 1/52
(d) 1/36

15) Arun draws 3 balls at random from a basket which contains 4 red
and 5 blue balls. What are the odds in favor of these being all blue?
(a) 5: 42
(b) 5: 37
(c) 4: 43
(d) 4: 50

16) 12 people, including the MD, the CFO, and 10 executives are to be
seated around a circular table. Find the probability that there are at
least 3 executives sitting between the MD and the CFO
(a) 5/10
(b) 3/5
(c) 1/3
(d) 5/11

17) In a single throw of 2 dice, the odds against drawing 7 are?


(a) 1: 5
(b) 2: 5
(c) 1: 6
(d) 5: 1

18) The integers p and q are chosen at random between 1 and 100, then
the probability that a number of the form 7p+7q is divisible by 5 is?
(a) 1/4
(b) 1/2
(c) 1/5
(d) 3/4

19) A wooden cask contains some tools i.e. exactly 12 nuts and 24
bolts. 1/3rd of each of these are defective. If 2 tools are picked up one
after the other without replacement, then what is the probability that
both are not defective?
(a) 45/106
(b) 23/135
(c) 46/105
(d) 89/105

20) Two squares are chosen at random from the small squares drawn
on a chessboard. What is the chance that the two squares chosen have
exactly one corner in common?
(a) 0.052
(b) 0.042
(c) 0.048
(d) 0.075

21) If the XAT center of 4 students can be any one of the 7 cities, then
calculate the probability that all the 4 students get any one of exactly 2
centers.
(a) 3/49
(b) 6/49
(c) 7/48
(d) 12/39

22) 6 boys and 3 girls are randomly placed in a row. Determine the
probability of no two girls being placed adjacently.
(a) 7/23
(b) 7/24
(c) 8/23
(d) 9/24
23) What is the probability that product of two integers chosen at
random has the same unit’s digit as the integers themselves?
(a) 3/10
(b) 1/25
(c) 4/15
(d) 7/15
24) A man speaks truth 3 out of 4 times. He throws a die and reports
that it is a six. Find the probability that it is actually a six.
(a) 2/3
(b) 3/4
(c) 1/8
(d) 2/7

25) An experiment succeeds twice as often as it fails. What is the


probability that in the next 5 trials there will be four successes?
(a) 82/232
(b) 17/53
(c) 80/213
(d) 80/243

Probability Solutions
Solutions for probability questions are given here. The solutions are explanatory,
detailed and easily understandable to help the candidates muster the topic effectively.

Test Your Probability Skills – Solutions


1) Option (d)
Probability of Getting an A – 10/50 = 1/5
Probability of getting a D – 30/49
Probability of getting a D – 29/48
Probability of getting a S – 10/47
Total probability= 1/5×30/49×29/48×10/47.

2) Option (c)
Probability of seating at least 2 children = 1- (Probability that the table seats
none or exactly one child)
5 people can be selected out of 10 in 10C5 ways = 252
The number of ways of seating a table of 5 with 0 children= 0 (only 4 adults)
Number of ways of seating a table with exactly one child= 6C1 =6 ways Required
probability = 1 – 6/252 = 41/42.

3) Option (c)
If A is selected, then two more students need to be selected. This can be done
in 5C2 ways= 10 If C is also selected, only one more out of the remaining 4 needs
to be selected. That can be done in 4 ways. Probability = 4/10 = 0.4.

4) Option (c)
Probability of a success= 0.3 Probability of a failure= 0.7 Required Probability
= 0.3 × 0.3 × 0.7 × 3C1 = 0.189.

5) Option (b)
Total sample space is 30 The number of multiples of 3 are 10 and the number
of multiples of 13 are 2, out of which none of them are common. Hence, the
answer is 2/5.

6) Option (d)
The probability of getting no six is (5/6)3 = 125/216. Hence the probability of
getting at least one six is 1 – 125/216 = 91/216.

7) Option (d)
Sample space is 90 numbers, of which , 90/3 = 30 are multiples of 3. 90/5 = 18
are multiples of 5, of which every 1 out of 3 are multiples of both 3 and 5.
Probability that a two-digit number is a multiple of 3 and not a multiple of 5 =
30 – 6 = 24/90 = 4/15.

8) Option b
First find the probability of sum being greater than 10.. Combination whose
sum of 12 is (6, 6) Combinations whose sum of 11 is (5, 6), (6, 5). Therefore,
there are totally 3 occurrences out of 36 occurrences that go against the given
condition. Probability whose sum of two numbers is greater than or equal to 11
= 3/36 = 1/12. Hence probability whose sum of two numbers is lesser than 11
= 1 – 1/12 = 11/12. Option (b) is the answer.
9) Option (a)
Lets consider upto the 9th apple. The first 8 apples should have 3 rotten ones
and remaining 5 good ones. This can be chosen in 4C3×11C5 ways. Total number
of selections of 8 apples out of 15 apples is 15C8. The last apple is the only rotten
one left, which can be selected in 1 way. Total number of ways of selecting that
1 apple from remaining 15 – 8 = 7 apples = 7C1 ways Total probability
= 4C3×11C5/15C8×1/7C1

10) Option (c)


If he picks any of the three balls, invariably any two of them should be of the
same colour. Hence the probability is 1.

11) Option (c)


The total possibilities are 2 × 2 × 2 = 8 (each customer has two possibilities).
These are AAA, AAB, ABA, BAA, BBA, BAB, ABB, BBB. The favourable outcomes
are only 4 (AAA, AAB, ABA, BAA). Thus the probability = 4/8 = 1/2.

12) Option (d)


No matter what sign Arpit throws, there is one sign Rita could throw that would
beat it, one that would tie, and one that would lose. Rita is equally likely to
throw any one of the three signs.
Therefore, the Probability that Arpit will win is 1/3.
Probability that Rita will win = 1/3
Probability of a tie = 1/3 .
Probability that Arpit will win = 1/3.

13) Option (c)


Probability of getting an even number of the first dice and the second dice is =
3/6 × 3/6 = 1/4.

14) Option (c)


The maximum total possible = 18
There are two options for Vijay 1) A + B + C = 17
Possibilities = 6, 6, 5
Number of combinations = 3!/2! = 3 2) A + B + C = 18
Possibilities = 6, 6, 6
Number of combinations = 1
Required Probability = 4/216 = 1/52.

15) Option (b)


Probability of all three balls being blue = 5_(C_(3 ) )/9_(C_(3 ) ) = 5/52 Odds in
favour of being blue = 5/((42-5) ) = 5 : 37.

16) Option (d)

From the diagram, suppose the MD is positioned at say, point 2, then the CFO
can be positioned at any of the points 6, 7, 8, 9, 10 Number of favourable
positions = 5. Total number of available positions = 11 Thus the required
probability = 5/11.

17) Option (d)


7 can be drawn in the following ways? 1 + 6, 2 + 5, 3 + 4, 6 + 1, 5 + 2 and 4 + 3.
Total number of ways = 6
Total number of possibilities = 6 × 6
Probability of not drawing a seven = ((36-6))/36 = 30/36
Odds against drawing a 7 is = 30 : 6 or 5 : 1.
18) The power cycle of 7 is 7,9,3,1
Thus, the only sums possible of 7p+7q is 2,4,6,0,8.
Only when the units digit is 0, is the expression divisible by 5. Hence, the
required probability is 1/5.

19) Option (c)


Out of the 12 nuts, 4 are defective and out of the 24 bolts, 8 are defective the
probability of picking up both non-defective tools
= 24/36 × 23/135 = 49/105.

20) Option (c)


If the first square chosen is one of the 4 corner squares, the second square can
be chosen in 1 way = 4×1.
If the first square chosen is one of the squares on the sides (other than corners)
= 24,
the second square can be chosen in 2 ways = 24 × 2
If the first square is any of the middle squares = 36,
the second square can be chosen in 4 ways = 36 × 4.
Total number of ways = 4 + 48 + 144 = 196
Number of ways in which 2 random squares can be selected in a chess board =
64 × 63
Required probability = 196/((64 ×63) ) = 0.048.

21) Option (b)


Total number of ways in which centres can be allocated = 74. Two centres can
be chosen in 7C2 ways. The number of ways they can accommodate 4 students
= 24 But this includes the two cases where one centre is having all the four
students. So, no. of ways = 24 – 2 = 14. So, total number of favourable ways =
7C2 × 14 So, probability = 7C2 × 14/74 = 6/49.

22) Ooption(b)
The number of ways that no two girls are placed together is the number of ways
in which 3 places marked with G are selected out of the SEVEN places. This can
be done in 7C3ways. Total no. of ways in which balls can be arranged = 10!/7!
×3!= 10C3. So, required probability = 7C3/10C3=7/24.

23) Option (b)


An integer can end with any of the ten’s digits (0, 1, 2 … 9) out of which if it ends
with one of the four (0, 1, 5, 6), the required condition will be satisfied.
The probability of an integer ending with 0 or 1 or 5 or 6 is 4/10=2/5.
Now the probability of second integer also ending with the digit that has come
in the unit’s place of the first integer is 1/10.
Therefore, the required probability = (2/5)×(1/10)=1/25.

24) Option (c)


For it to be an actual six two things are required to happen together.
1. He is speaking the truth. Let this be an event A.
2. The die shows 6. Let this be the event B.
We have: P(A) = 3/4 P(B) = 1/6
The probability that both events happen together = P(A) × P(B) = 3/4 × 1/6 =
1/8.

25) Option (d)


An experiment succeeds twice as often as it fails. So, the probability of success
is 2/3 and the probability of failure is1/3.
In the next 5 trials, the experiment needs to succeed in 4 out of the 5 trials. 4
out of 5 trials can be selected in 5C4 = 5 ways.
So, required probability = 5 × (2/3)4× (1/3) = 80/243.

CAT Data Interpretation Questions


The CAT exam includes the section of Data Interpretation to test the problem-solving
and interpretation skills of the candidates. In general, the CAT data interpretation
questions are highly scoring and easily solvable. The MBA aspirants are expected to
solve more number of CAT DI questions to increase their CAT score and get admissions
to any of the prestigious B-Schools in the country. So, here are some practice data
interpretation questions for CAT that might help the candidates to get confident in the
Data Interpretation section and score well in the exam.

Data Interpretation Questions for CAT


Directions for questions:

Read the following passage and solve the questions based on it.

CASELET 1:

Mr X has built a mansion with 10 rooms. He was confused about the colours he should
use while painting each room. He had the following choice of colours: blue, hazy grey,
jumping yellow, teal, violet latte, Terry Cherry and happy pink. It was also known that
he could paint more than 1 room with a single colour. Finally, he set up an algorithm to
decide the colours that he would be using.

(i) If he painted any room teal, then he did not paint any other room happy pink.

(ii) If he painted any room blue, then he did not paint any other room jumping yellow.
(iii) If he painted any room blue, then he painted at least one room happy pink.

(iv) If he painted any room jumping yellow, then he painted at least one room violet
latte.

(v) If he painted any room violet latte, then he painted at least one room happy pink.

(vi) if he painted any room happy pink, then he painted at least rooms happy pink.

1) Which one of the following could be a complete list of the number of rooms and
colours that Mr X used to paint some of the rooms of his house?

a) one blue, one Terry cherry, one violet latte, two happy pink

b) one blue, one teal, one Terry cherry, three happy pink

c) two blue, one teal, three Terry Cherry

d) one jacket, one Terry cherry, two violet latte and one happy pink

2) If Mr. X did not paint any room happy pink, what was the maximum number of
the different types of colours that he could paint ?

a) two

b) three

c) four

d) five

3) Which one of the following statements must be false?

a) Mr. X painted exactly four rooms with colours, one of which was a hat.

b) Mr. X painted exactly three rooms with colours, one of which was a happy pink.

c) Mr. X painted exactly four rooms with colours, one of which was a blue.

d) None of these

4) If Mr. X painted as many different types of colours as possible, then it must be


true that he did not paint one of the following types of colours.

a) blue

b) hazy grey

c) teal

d) jumping yellow
5) If Mr. X painted at least one room, find out which one of the following are the
minimum and the maximum numbers of the types of colours that he could paint ?

a) 1, 4

b) 1, 5

c) 1, 6

d) 2, 5

CASELET 2

It’s Valentine’s Day and five boys Amit, Bhuvan, Chetan, Dilip and Ehsaan are buying
flowers for their respective girlfriends.

Each of these boys has a preference from 1 to 5 (1 being first preference, it is given the
first rank) of flowers among orchid, rose, carnation, gerbera and daffodil.

There is a level of dissimilarity between the five boys and this is measured as the sum of
the differences in the ranks assigned by them to each of these 5 flowers. The greater this
difference, the more dissimilar the persons.

The following table indicates the preferences of each of these five boys

6) The pair of persons who are the most dissimilar among the following is:

a) Amit- Bhuvan

b) Bhuvan- Dilip

c) Dilip- Ehsaan

d) Chetan- Ehsaan

7) Who among the following is most similar to Amit?

a) Bhuvan

b) Chetan

c) Dilip

d) Ehsaan
8) Which of the following pairs are the least dissimilar among the five?

a) Dilip-Ehsaan

b) Amit- Chetan

c) Chetan-Ehsaan

d) Amit- Ehsaan

9) For the person who’s second preference is Rose, what is the level of
dissimilarity with the one who’s fourth preference is orchid?

a) 8

b) 6

c) 10

d) 12

10) What is the level of dissimilarity between Amit and Ehsaan?

a) 6

b) 8

c) 4

d) 10

CASELET 3

The two pie charts below show the percentage market share on value basis of the
companies A to D and others in a sectorial market for 1999 and 2000

FIGURE 1: YEAR 1999 FIGURE 2 : YEAR 2000

Market size 1999= 150 crore Market size 2000= 375 crore
11) Which company had the minimum growth in sales in these two years?

a) B

b) A

c) C

d) Others

12) If each company increases its sales value by 10%, then what is the percentage
growth of the detergent market?

a) 10%

b) 20%

c) 30%

d) 8%

13) If the total sales of the market is doubled for 1999 and 2000, what would be
the ratio of sales of D for 2000 to 1999?

a) 2:1

b) 3:4

c) 15:8

d) 10:3

Additional data for questions 14 to 16

Product share of sales turnover for A in 1990

14) What is the market share of Bros in the market in 1999?

a) 7.5%

b) 12%
c) 9%

d) cannot be determined

15) Which company has the highest profit percentage in 2000?

a)A

b) B

c) C

d) cannot be determined

16) If the expenditure of C is growing at the rate of 10% per annum, what is the
profit percentage in 2000 (Assume profit =25%)?

a) 404%

b) 304%

c) 379%

d) 275%

CASELET 4

On a playing ground, Ariya, Amita, Binoy, Shama and Payal are standing as described
below facing the North.

(i) Amita is 40 metres to the right of Shama.

(ii) Ariya is 60 metres to the south of Amita.

(iii) Binoy is 25 metres to the west of Shama.

(iv) Payal is 90 metres to the north of Ariya.

17) Who is to the north-east of the person who is to the left of Amita ?

a) Binoy

b)Shama

c)Ariya

d)None of these

18) If a boy walks from Binoy, meets Shama followed by Amita, Ariya and then
Payal, how many metres has he walked if he has travelled the straight distance all
through?
a) 215 metres

b)185 metres

c) 155 metres

d) 245 metres

19) Who is to the south of the person who is to the north-east of Shama?

a) Ariya

b) Binoy

c) Amita

d) both a &c

CASELET 5

Eight years ago, Yellow was half as old as Green will be when Green is one year older
than Blue will be at the time when Yellow will be five times as old as Blue will be 2 years
from now.

Ten years from now Blue will be twice as old as Green was when Yellow was nine times
as old as Blue.

When Blue was one year old, Yellow was three years older than Blue will be when Green
is three times as old as Yellow was six years before the time

when Green was half as old as Blue will be when Yellow will be ten years older than
Yellow was when Green was 1/3 rd as old as Blue will be when Yellow will be three
times as old as she was when Green was born.

20) How old is Blue?

a) 4

b) 6

c) 2

d) none of these

21) How old will Green be 10 years from now?

a) 17

b) 8

c) 18
d) none of these

22) How old would have Yellow been 6 years ago?

a) 15

b) 8

c) 9

d) none of these

CASELET 6

After the 2nd MOCK CSAT, Seven friends – Charles, David, Hanish, Kedar, Mahoud, Ninja
and Raul are comparing their scores in this exam. We know the following information
about their scores.

(a) All of them had distinct scores.

(b) Kedar scored the same marks as the average of the marks scored by Charles and
David where Charles scored more marks than David.

(c) Both Mahoud and Ninja scored less marks than Hanish but more than Raul and the
marks scored by Raul is not the least

(d) The number of persons who scored more marks than Kedar is same as the number
of persons who scored less marks than Kedar.

(e) Charles scored less marks than Mahoud.

23) Among them who scored the second highest marks?

a) Ninja

b) Mahoud

c) Chandru

d) Cannot be determined

24) Among them who scored the second lowest marks?

a) Charles

b) Raul

c) Ninja

d) Cannot be determined

25) How many people scored more marks than Charles?


a) 5

b) 4

c) 3

d) 2

26) Whatis the number of persons whose scores are in between the scores of
Ninja and David?

a) 3

b) 2

c) 1 D) 0

d) 0

27) Which of the following is true?

a) Ninja scored more than Mahoud.

b) David scored more than Ninja.

c) Raul scored more than Kedar

d) Ninja scored more than David.

CASELET 7

The given figure shows the production and consumption of Ragi in India over a period
of 5 years.

28) If surplus ragi available each year was exported, what % of the ragi produced
between the years ’97-98 and ’00-01 was exported?

a) 15%

b) 10%
c) 6%

d) none of these

29) Between the years ’96-97 and ‘00-01,the following can be said about the
cumulative production and consumption of ragi

a) Cumulative production of ragi exceeded that of consumption by 18 lac tones

b) Cumulative consumption of ragi was 89% of the cumulative consumption of ragi


during this period.

c) Cumulative production of ragi exceeded cumulative consumption of ragi by 4.7%


during this period.

d) Consumption of ragi never exceeded the production of ragi during this period.

30) Which of the following statements are true?

I) The YOY rate of growth of production of ragi has been greater than the YOY rate
of growth of consumption of ragi during the period 97-98 to 00-01

II) The CAGR rate of growth of productionof ragi has been greater than the CAGR
rate of growth of consumption of ragi during the period 97-98 to 00-01

III) The amount of ragi exported in a given year was greater than the previous
year during all the years in the period 97-98 to 00-01

a) I only

b) I and II only

c) III only

d) II only

31) What was the % rate of growth in production of ragi between the period 99-
00 and 00-01?

a) 4.05%

b) 5.71%

c) 1.67%

d)10%

32) Which of the following years witnessed a two digit rate of growth of ragi? I)
97-98 II) 99-00 III) 00-01

a) I only
b) I and II

c) I and III

d) none of these

CASELET 8

The International Kabbadi League (IKL) was formed last month to give a boost to the
game at international standards.

It had a tournament, where 2 teams played some matches. Each team comprised of 7
players each.

The listings of the 2 teams X and Y were lost, but certain details regarding the players
were available. A,B,C,D,E,F,G,H,I,J,K,L,M and N are the players.

D and E were in Team X, K and G were in team Y.

H and B were in the same team, but not in the team in which F was.

The sum of the scores of members of Team Y was not greater than 115.

The table containing the details of the players and their scores is below

33) Which of these players was definitely in Team Y?

a) L

b) M

c) N

d) None of these

34) If the score for team Y was less than 110, what could be the score of team X?

a) 135

b) 137

c) 139

d) cannot be determined

35) Which of these players could not be in team Y, if the score of Y was 115?

a) A
b) L

c) M

d) N

36) Which of these players was definitely in team X, if the score of Y was 112?

a) I

b) L

c) M

d) N

37) Which of these players are definitely in Team X?

a) F

b) H

c) B

d) none of these

CASELET 9

To assess the SOPS handed out in the Union budget during a prime time program, a
news channel must choose 2 GDA members and 2 RGP members. At least one should be
an economist and at least one should be an industrialist. The GDA members are A,B,C,D
and E, RGP members are F,G,H and I. C,F, and G are economists. D and I are
industrialists. F and C are at loggerheads, and will not appear together. F will take part
only if A takes part. D refuses to participate if G is present and E refuses to participate if
I is present.

38) Which of the following is not an acceptable panel?

a) F,H,A,D

b) G,H,A,C

c) H,I,B,C

d) F,I,A,D

39) How many acceptable panels can be put together?

a) 9

b) 5
c) 10

d) 11

40) Which of the nine members in the panel will feature in the greatest number of
different acceptable panels?

a) C

b) F

c) A

d) I

41) If A and B are chosen as the GDA members, then who will represent the panel
from the RGP party?

a) F and I only

b) G and H

c) G and I only

d) a or c

CASELET 10

Six players, Kramnik, Topalov, Shirov, Leko, Polgar and Anand participate in a chess
tournament.

In the first round, each player plays one match against every other player. The winning
player is awarded 3 points and the losing player gets 1 point.

In case of a draw, each player is awarded 2 points. The player with the highest number
of points enters the final.

The semifinal is played between the next two players. The winner of the semifinal
enters the final. The winner of the finals is declared the champion.

There can be no draws in the final and the semifinal. The results of all the matches
played by the players at the end of the tournament are given below.
42) Who is the champion?

a) Kramnik

b) Topalov

c) Leko

d)Anand

43) The semifinal is played between players

a) Kramnik and Topalov

b) Topalov and Polgar

c) Kramnik and Anand

d) Polgar and Anand

44) Find the points of the semifinalists before the semifinal

a) 9,10

b) 10,10

c)10,11

d)11,11

45) Which two players played the final?

a) Kramnik and Topalov

b) Topalov and Leko

c) Kramnik and Anand

d)Topalov and Anand

46) Which of the following is/are true?


I. The top three rankings at the end of the tournament are the same as those at the
end of the first round.

II. Anand won the maximum number of matches in the first round.

III. Kramnik has the highest number of points at the end of first round.

a) I only

b) II only

c) III only

d) I, II and III

CASELET 11

The Poolside chess club management had misplaced the order of it’s annual winners
from 2001 to 2005, among Pavan, Bishu, Rahul, Arti and Gavin.

When 5 of the regular staff (A,B,C,D,E) in the club were questioned about the winners,
each gave their respective orders as shown in the table.

When the records were finally retrieved, the management, out of sheer exuberance,
decided to reward the staff as follows. If any of the staff had named all 5 in the right
order he would get Rs 10000 as cash prize. If the staff names “n” out of all 5 years
correctly, he will get (n+1) thousand Rupees as cash. It was found that each staff won a
different amount of money

47) Who won the chess tournament in 2003?

a) Pavan

b) Gavin

c) Arti

d) none of these

48) How many of the staff correctly mentioned the winner of 2003 ?

a) 1

b) 4

c) 0

d) 3

49) Who won the least amount as reward?


a)A

b) D

c) B

d) C

50) Who won the chess tournament in 2001?

a) Arti

b) Rahul

c) Bishu

d) cannot be determined

51) How much was the middle amount won as reward money?

a) 2000

b) 4000

c) 1000

d) none of these

CASELET 12

Students have applied for a graduation programme to the following universities in the
U.S. through a centralized admission process for foreign education : Carnegie Mellin
University(CMU), Georgia Tech University(GTu), Penn State University(PSU), Michigan
Ann Harbour(MAH), Cornell(COR), University of Pensylvania(U-Pen), Michigan State
University(MSU), Indianapolis Sate University(ISU), and California State
University(CSU).A list of eleven short-listed students has been forwarded to all the
universities. Each university has prepared a consolidated list based on the following
guidelines:

The percentage of marks obtained in the 10th, 11th and 12th are counted as points for
the respective exams.

First, second and third preferences have been given 15, 10 and 5 points respectively.
Points for the SAT test are equal to the equivalent score given.

If a candidate has done project work at the national level, he/she will be awarded 5
additional points.

Only one applicant can be selected for each university.


An applicant can take admission only in a university to which he has given preference.

If a candidate is eligible for more than one university, he will be chosen by the order of
preference.

The candidate with the highest points is given priority for getting admission to a
university.

If two or more candidates have the same points, the person who has given higher

preference to the University will be given priority and if that is also the same then the
person who has done a national level project will be given priority.

52) Iqbal has gained admission to which university?

a) COR

b) GTU

c) ISU

d)None of these

53) What is the total number of candidates who have gained admission to
universities in the U.S.?

a) 8

b) 9
c) 10

d) 11

54) Who got admission to U-Pen?

a) Kabir

b) Gulnashin

c) Farah

d) Badshah

55) How many candidates have gained admission to a university which was not
their first preference?

a) 0

b) 1

c) 2

d) 3

56) How many universities were left without eligible candidates?

a) 0

b) 1

c) 2

d) 3

CASELET 13

In a standard 5 class, the students are arranged according to the increasing order of
their marks in an examination. The marks obtained by every student, starting from the
third student, are the sum of the marks obtained by the earlier two students.

57) If the 12th student got 809 marks and the 2nd student got 6 marks, then what
is the sum of the marks obtained by the first ten students?

a) 815

b) 803

c) 800

d) Cannot be determined
58) If the 3rd student got 10 marks and 5th student got 26 marks, then how many
marks does the 12th student get?

a) 754

b) 466

c) 1220

d) Cannot be determined

CASELET 14

Each of the five friends – A, B, C, D and E met at a party. They live in different cities
among – Delhi, Mumbai, Bangalore, Chennai and Hyderabad and work in different
companies among – Infotec, Quetec, Rototec, Simotec and Tetratec, not necessarily in
the same order. We know the following information about them.

(a) A lives in Delhi but does not work in Quetec.

(b) C works in Tetratec.

(c) The person, who lives in Bangalore, works in Simotec

(d) Neither D nor B lives in Chennai.

(e) B works in Infotec and E lives in Hyderabad.

59) Who lives in Mumbai?

a) B

b) D

c) E

d) Either (A) or (B)

60) Which company does D work in?

a) Quetec

b) Rototec

c) Simotec

d) Either (A) or (B)

61) In which city does the person who works in Tetratec live?

a) Delhi
b) Chennai

c) Hyderabad

d) Either (1) or (2)

62) In which city does the person who works in Infotec live?

a) Chennai

b) Delhi

c) Mumbai

d) Either (1) or (2)

63) For which company does the person who lives in Hyderabad work?

a) Quetec

b) Infotec

c) Tetratec

d) Rototec

CASELET 15

4 friends; Karan, Lavanya, Mariam and Nargis are playing a game called “Lose It”
wherein the loser doubles the money of each of the other players. They played 4 times
and each friend lost one game in the alphabetical order. Each friend has Rs 96 at the end
of the last game.

64) Who had same amounts at the end of the second game?

a) Mariam, Nargis

b) Karan, Nargis

c) Karan, Lavanya

d) none of these

65) How much money did Mariam have initially?

a) 25

b) 54

c) 122

d) none of these
66) Who had the minimum deviation from the opening amount?

a) Karan

b) Lavanya

c) Mariam

d) Nargis

67) What was the amount left with Karan at the end of the second round?

a) 6

b) 12

c) 18

d) 24

68) Who had the maximum profit by the end of the 4th round?

a) Karan

b) Lavanya

c) Mariam

d) Nargis

69) Eight students J, K, L, M, N, O, P and Q go on for a science exhibition in two


batches of four each. J and K never go together. M and O do not go together. K and
Q have to go together. If K and L go in one batch, then which of the following can
be in the other batch?

a) A. J, M, O and P

b) J, M, Q and P

c) J, M, N and P

d) J, L, N and P

70) Three Students – Jose, Bharath and Simon – have to select three sports each
out of the six sports– Hockey, Baseball, Tennis, Basketball, Cricket and Football. If
Football is selected, then Hockey cannot be selected. If Simon selects Tennis, then
Jose does not select Football. If Bharath selects Baseball, then Simon cannot select
Tennis and if Jose selects Hockey, then Simon selects Cricket and Football. If
Simon selects Football and Bharath selects Baseball, then Simon cannot select

a) Hockey and Tennis.


b) Baseball and Tennis.

c) Tennis and Basketball.

d) Hockey and Cricket

Data Interpretation Solutions


The solutions of Data Interpretation are explained here. To get the Questions, click CAT
Data Interpretation questions.
Solutions for Data Interpretation

1) Option (a)
Solution:
Option (b) is a violation of condition 1.
Option (c) is violation of condition 3.
Option (d) is violation of condition 6.
The correct answer is option a.

2) Ans. (b)
Solution: Since he did not paint any happy pink => he won’t be painting any
room blue ( condition 3) and he also won’t paint any room violet latte
(condition 5) => he won’t paint any room jumping yellow (condition 4). So, he
can’t paint 4 types of colours out of 7 => he can paint 3 different types
of colours. The correct answer is b.

3) Ans. (d)
Solution: if Mr. X paints a violet latte => he must paint happy pink that too 2
rooms (condition 5 and 6).
If Mr. X paints happy pink => he will paint 2 rooms with it (condition 6) and he
can paint 1 violet latte.
If Mr. X paints a room blue => he cannot paint jumping yellow (condition 2) , he
painted happy pink that too two of them (condition 3 and 6).
He also could have painted a room hazy grey, terry cherry or violet latte as these
two have got no constraints on them.
So, option (d) can also be true.
Hence none of these is the answer.

4) Ans. (c)
Solution: From the solution of the 2ns question, he should paint happy pink if
he has to paint the maximum colours possible => he cannot paint teal (
condition 1). the correct answer is c.

5) Ans. (b)
Solution: He can paint hazy grey and then he will paint the minimum number
of colours 1 as there are no constraints on him for painting hazy grey.
If he paints happy pink, he cannot paint teal and he can paint any one of jumping
yellow or blue (condition 1 and 2). =>maximum number of colours that he can
paint = 5. Correct answer is (b).

6) Option (a)
Solution: Go from the answer options

Hence answer= (a).

7) Based on the table below

8) Go from answer options

9) The person who’s second preference is Rose is Bhuvan and the one who’s
fourth preference is orchid is Dilip.
Their level of dissimilarity is 8 (from table 1)

10) From table two, the answer is 8


11) Option(c)

Growth in sales of B = 63.75


Growth in sales of A= 52.5
Growth in sales of C= 26.25
Growth in sales of B= 30
So option (c).

12) It will be equal to that of the whole market = 10%

13) The ratio will not change because if the total sales are doubled, the sales of
D will also double for both the years.
Hence the ratio will be 75/22.5=10/3

14) Bros = 30% of A


A= 30% of detergent market
Bros= 30% of (30% of market)=9% of market.

15) To determine profit percentage, both sales and expenditure is required. As


expenditure data is not available, the profit percentage cannot be determined

16) Cost of C = Sale/(100+profit percentage) x 100 = 150×15%/1.25 = 18 crore


C’s cost in 2000= 20
C’s sale in 2000= 75
Profit percentage= 55/20 x 100= 275
Solutions questions 17-19
17) To the left of Amita is Binoy and Shama, north east of both is Payal.

18) B-S(25) S-AM(40) AM-ARIYA(60) ARIYA-P(90) TOTAL=215


19) Northeast of Shama- Payal South of Payal- Amita and Ariya

Solutions questions 20-22


This should be solved either by assumption using answer options or by forming
equations. Since none of these is also involved, its better to go by using
equations
Let y1, y2 … be the indefinite years. Let t be the year blue was born, j green and
m- yellow.
Let y be the current year y – 8 – m = ½ ( y2- j) = 1+y3 – ty3 – m = 5( y+2-t)—
(1)
y+10 –t= 2( y1-j) y1 –m= 9(y1 –t)————(2)
t+1-m = 3+ y4 – ty4 –j = 3(y5 – 6 – m )y5 –j = ½ (y6 – t) y6 – m =10 + y7 – my7
– j = 1/3 (y8 – t) y8 – m = 3(j-m)
t= y-3j = y-8m = y-15
Therefore, green= 8 years, blue= 3 years & yellow = 15 years

Solutions questions 23-27


Let the marks scored by Charles, David, Hanish, Kedar, Mahoud, Ninja and Raul
be represented by C, D, H, K, M, N and R respectively.
From (a), all scored distinct marks:
From (b), C > K > D
From (c), H > M, N > R
From (d), Kedar scored the fourth highest marks.
From (e), M > C
From (c), R is not the least marks scored.
∴ D is the least.
From (b), (c) and (e), H > M > C > K > D
As K is the fourth highest score and D the lowest score and as N > R, N and R
must be the fifth and sixth highest scores respectively.
H>M>C>K>N>R>D

23) Mahoud scored the second highest marks.


So, Option (b)

24) Raul scored the second lowest marks.


So, Option (b)

25) Only Hanish and Mahoud scored more marks than Charles.
So, Option (d)

26) Only Raul scored in between the scores of Ninja and David.
So, Option (c)

27) Only Option (d) is true.


So, Option (d)

28) Total ragi surplus between 98-98 to 00-01 = 6+9+8+12 = 35 lakh tones
Total ragi produced between 97-98 and 00-01 = 132 + 138 +140 + 148 = 558
lakh tones
Percentage of production exported = 35/558 x 100= 6.27%

29) Cumulative production of ragi= 678 lakh tones


Cumulative consumption of rice = 647 lakh tones
Cumulative production exceeds cumulative consumption by 4.7%

30)
From the table, we can see that not in all years in the YOY% growth of
production greater that YOY% growth in consumption.
Hence, statement I is not true Overall production has grown faster than
consumption. Hence, statement II is correct
In 99-00, surplus ragi was less than that in 98-99. hence statement III is also
not true. Answer= d

31) Production increased from 140 lakh tones to 148 lakh tones, increase
8/140 = 5.71%

32) The year 97-98 alone witnessed a 2 digit rate of growth in production
Solutions questions 33-37
There are 2 cases
(1) D, E, H, B in team X and K, G, F in team Y (total = 40)
So the only possible team Y is K, G, F, L, M, N and I (total=115)
(2) D, E, F in team X and K, G, H, B in team Y (total = 55)
So the possible additions to Y are
(a) L, M and N (total = 53)
(b) L, M and I (total = 60)
(c) I, M and N (total = 55)
(d) I, L and N (total = 57)

33) (d).
34) (b).
35) (a).
36) (c).
37) (d).

38) Option B has no industrialist.

39) (FHAD) (HIBC) (FIAD) (AFDI) (ABFI) (ACHI) (CDHI) (BCGI) (CEDH) – 9
panels

40) I appear on 7 panels.


41) A and B are neither economists or industrialists. Hence there has to be an
economist and an industrialist from RGP if
F is selected➜ I can be selected
if G is selected ➜
I can be selected hence option d Solutions questions 42-46 Tabulate the total
number of points obtained by each person

From the data in the table, it can be clearly inferred that Anand wins the
tournament. Also the semi-finalists and the finalist can be deduced as K, P and
A.
Also, one definite player of the semis is P, and it can also be said that P loses in
the semis. Therefore the finals will be played between K and A.
We can directly mark the answers to questions 42-d and 45)-d
If A won the tournament, then he will have a score of 14-3=11 before the finals.
Notice that K has only one loss and that loss is the loss in the finals, which
implies that K has won all the tournaments before.
Before the finals, A’s score will stand at 14, P’s score at 11 ( he gains one point
after losing the semis) and K’s score will stand at 13, who will be the top scorer
and will directly go to the finals.
From this, we can clearly say that A will play the semis against P.
A’s score then becomes 11+3=14 and P’s score 10+1=11
With this we can answer questions 43)- d and 44)-c also, 46) option (c)
Only statement 3 is true as K has only one loss, which occurs in the finals

Solutions for questions 47-51


Since there are 5 staff members, if one gets the correct order in which 4 players
won the tournament, then definitely the order for all 5 players will be correct.
Since it is given that all 5 staff got diff values, it must be 5,3,2,1,0
Assume that Staff A got all of them correct then B,C,D, and E got 1,3,0,2 players
in the correct order.
Similarly, checking for other staff members, we arrive at two possibilities
47) Option (a).
48) Option (b).

49) In either arrangement, D did not get any of the players in the correct order.

50) Cannot be determined.

51) Since “2” of the winners being in the right order is the middle amount from
(5,3,2,1,0) the amount won = (2+1)thousand = 3000 rupees( Even if you are not
solving the entire caselet, you must solve this question as in any case the middle
value will be 3000)..

Solutions questions 52-56:


Calculate the total score by each student and the as total score obtained by each
student WRT each university as follows (including +5 for those who have done
a national level project)

Now calculate the score based on preference for each university


The final table can be deduced as follows

Solutions for questions 59 to 63:


From (a), (b) and (c),
as A lives in Delhi, he does not work in S
Also, he does not work in Q and T
From (v), B works in ‘I’ ∴ A works in R
From (d) and (e), none of D, B and E lives in Chennai.
∴ C lines in Chennai.
From (c) and (d), as B works in ‘I’,he does not live in Bangalore.
∴D lives in Bangalore and works in S.
E works in Q and B lives in Mumbai. The distribution table is be as follows
59) B lives in Mumbai. (A)

60) D works in Simotec. (C)

61) The person who works in Tetratec lives in Chennai.(B)

62) The person who works in Infotec lives in Mumbai.(C)

63) The person who lives in Hyderabad works in Quetec. (A)

Solutions questions 64-68:


Based on the information, we can start from the bottom-most row of the table
and fill it upwards as shown(The loser is highlighted)

So answers are (c), (b),(b) (d) and (d)

69) Two batches have to be formed from students J, K, L, M, N, O, P and Q. If K


and L are in one batch, then Q must be in that batch, as K and Q are together.
J cannot be in that batch as it is given that J and K cannot go together.
K, L, Q form a team of 3 members, then the fourth member may be M or N or O
or P.
As M and O do not go together, either M alone or O alone should go with K, L, Q.
So the other batch may be either “J, N, O, P” or “J, M, N, P”. Option (C)

70) It is given that A. if Football is selected, then Hockey cannot be selected. B.


if Simon selects Tennis, then Jose does not select Football.
Hence if Jose selects Football, Simon cannot select Tennis. C. if Bharath selects
Baseball, then Simon cannot select Tennis.
Hence if Simon selects Tennis, Bharath cannot select Baseball.
D. if Jose selects Hockey, then Simon selects Cricket and Football.
Hence if Simon does not select Cricket or does not select Football, then Jose
cannot select Hockey.
Simon does not select Tennis, as Bharath selects Baseball. Further, if Football is
selected, Hockey cannot be selected.
Therefore, the sports that Simon cannot select now are Hockey and Tennis.
Option (A)
Stay tuned with Byju’s to get the latest updates on CAT Exam. Visit Byju’s to know the CAT
Syllabus, CAT Exam Pattern, etc.

You might also like